Você está na página 1de 530

Published by :

LAXMI PUBLICATIONS (P) LTD. 22, Golden House, Daryag~, New Delhi.llOO02.

{ OU·232823_88 Phones: .011-232823'10-

Faxes : {OU-23252572 011-23 26 22 79

Brcuu:hes:

• 12911, nlrd MaI.11. &ad,IX CI'OIlII, Chaninllpet; Banpiore(PltoM : 080-26 61 1561)

• .26, _Damodaran Street, ~ Nagar, CheJmaI !P~ : 044·24 84 47 26) .

• St. Benedid', &ad, CoclDD-(Pho,... : 1J484.239 70 04)

• Pan. Bazar, Rani Bari, G_ahati (Phollel : 0361·.21i4 36 69, 2!';1 38 81)

• 4 .• 2-453: bt Floor,·Ramk.ote, Hyduabad!P~ : Q40.U 76 02 47)

• Adda Tanda. Chow.k, N.D. 365, J~ City (Plio,... : OlBl-222 12 72)

• 106lA, lBt Floor, S.N. Banerjee Road, Kolkata (Pho.flU: 003-22 .27 37 73, 22 27 5247)

• 18, Madan Mohan Malviya Mug. Luclmow- (P~ :' 0522-220 95 78) • ,l28A, Block 3, FinltFloor, Noo~ BuiJdill&", L.J. Ro.ad:Mwnbai

(Plio,... : 022-24 46 39 98) • Radh. Govind Street, Tharpagn., Ranehi (Pho"" : 0651.230 77 64)

EMAIL: coiax.mi@hotmail.com

WEBSITE.:. www.laxmipuhlications.C?m

@ All-Rights Reserved w"ith the Author and Publishers.

MGA-6'116-176-G. SOLID GEOMETRY (e)

Price ; Rs. 175.00 Only

C-U073/OS/07

. tau, ~peHtli"ll41 : SpectrUm Media, New DeIhl Prinkd <II : Mehn. OI&et P"rMo. Ne';' Deihl

Urheberrechtllch qeschutztes Materia

CONTENTS
Page
Important Results and Formulae (i)
General Methods (xvii)
CIMlrdinal!lS
2. Direction Cosines and Projections IS
3 The, II:lane 54
4. The Straight Line 126
5. Two Lin.es, Intersection of Three Planes 200
6. Change of Axes 297
7.. The SEhere 315
8 The CQne 412
9. The Cylinder Sl1.I
10. The Conicoid 526 U rheberrec htl ich gcsc h utztes Materia

IMPORTANT RESULT AND FORMULAE TO BE REMEMBERED

CHAPTER!

1. C<HlrdiDates. IfP(x,y, z) is any point in space, then

x = PL :0 .1 distance of P from YZ plane.

y = PM = .1 distance of P from ZX plane.

z = PN = .1 distanee of P from XY plane.

z

z

P (x. Y.l)

l~_+--''__--(jp (x. Y. i)

z

a

x

o

A

x

Y N Y

Also if PA .l OX, then OA = x,

Similarly if PB .1 OY, then 08 "" y, and if PC .1 Oz, then OC = z,

In filet x, )'. z are the distances travelled respectively along x-axis, j-axls, z -axls respectively in order to reach .P(l', )" z) from (0, 0, 0).

1. Distance Formula. Distance between two points (XI' )'1' Zl) and (X2' )'" %1) and

d = J(X1 -xJ)z +(y. -Yz)z +(zi - zz)z.

3. Section Formula. If (x, )" z) be the point which divides the join of (XI' )'1' %1) and (x" )'" %J in the ratiom, : m, internally, then

:I.:: mlx1 + ml:l.. ml+ml mlYl+m2YI y=.

m.+mz

z=m1.~+mlzl ml+ml

r=-«>.

m,:m,

(x,. Y .. z,) (ll. Y. z) (x,. y,. z.J

Rule. Multiply ml by the X oo-ordinates of the point remote from m, and m, by the x co-ordinate of the point remote from m2 as shown by

U rhcberrec htl lch gesc h utztes M ateri a

(il)

arrows, add these products, and divide the sum by ml + mr This gives x c~ordinales of the point of section.

Similarly for the y and % co-ordinates.

In case of emma! division,

x = mlxl -~.cl m,-ml

y=mlYl-~YI m,-~

% '" ml%l -"':i%, m,-~

Mid-poiD. Formula. The mid-point of the join of (x,. Y" z ) and (XI' Yl' %1) is

and

(XI + .. ilL y, +Yl, z! +ZI)

2 '.2 1

( sum of x co-ordinates sum of yco-ordinales

i.e. 2':z . •

sum of % c;ordinales )

4. Centroid. The centroid of the ~ whose vertices are (XI' YI• %1)' (X1,y1• %1)' (Xl' YI, %1) is

(x,+XJ+II:) YI+Y2+Y3 Zl+~+~) 3 ' 3 ' .3

(sum of :r co-ordinates sum of yco-ordinates

l.e; .3' 3 •

sum of % C;Ordinoles )

CeDfroid of the tetrahedron whose vertices are (x, YI• %1)' (Xl' ,vI' %1)' (Xl' YJ• El) and (x~ Y •• E.> is given by

(XI +1.1+1:1 +11:4 YI+YJ+Y,+Y' Z.+%1+~+Z4)

4 • 4 • 4

(sum of oX co-ordinales sum of yco-ordinates

I.e. 4' 4 '

sum of % c;ordinalu )

U rheberrec htl lch gesc h utztes M ate,; a

(iiI)

Remember. The centroid of a 11 divides each median in the ratio 2 : I and the centroid of the tetrahedron divides the join of each vertex to the centroid of the opposite face in the ratio 3 : 1.

CHAPTER 2

I. If I, m, " are the direction-cosines (d.c. '5) of a. lin.e OP and OP '" r, then the co-ordinates of. P are (lr. Mr. or).

2. If I, m, " are the d.c.'! of a line, then 11 + 1111 + 01 = 1.

3. If a, b, c are the direction ratios (or proportional d,c, 's) of a line, then the actual d.c.'s are

abc

.Jr.a1 ' .lEal • .JIal where UT = Ql + fiZ + t?

Rule. To fmd the actual d.c.'s of a line whose proportional d.c. 's are a,

b, ewe divide each ofthcse by Jal +b1 Tel. then the resulting ratios

Q b c

.Jra:' .Jr.a:' Jr.ti are the actual d.c.'s.

4. Direction roslDes of the join of two points (XI' YI, ZI) and (Xl' Y1, %1) are proportional to (Xl - XI' Y1 - YI, .~ - ZI).

I. e. difference of X co-ordinates, difference of Y co-ordinates.

diffirence of z co-ordinates.

S. Lagrange's Identity. If II' ml, nl and II' ml, nl are the two sets of three numbers (nol necessarily dc .. 's), then

(I: + mf +nh(l~ +m~ Tab -(1111 + mlml +0101)1 = (m101 - 0lm1i +(0111 -1101)1 +(llml- mllz)!

6. Aotle Formulae

(I) If II' ml, "1 and 11' mI' "1 are the actual d.c.'s of two lines, then the angle e between the two lines is given by

ees e '" II~ + mlma + DID1. - III~.

sin 8 = J(miDz -Dlm!)1 +(DI12-lln1)2 +(llml-IZm,)l = Jr.{mlD1 - Dlml)Z

tan e = sin e

cosO

= JI(mID 1- Dlm2 )z IIllz

U rhcberrec htl lch ge5c h utztes M ateri a

(i'v)

(1) If a,• b, c, and a1• hI' c2 are tbcproportionaJ d.c, 's of two lines in.c.lined at an angle e, then

and

For complete rormula, use ± sign with the radical sign in the formula for sine and tangent.

7. Condition of perpendicularity of two lines whose d.c.'s are I" mi' n, and 11' m,• n1 (or proportional d.c.'s a,• b, c, and a,• b,. e,) is· .

II~ + mimI + niDI = 0 (or .'.1 + bib: + C'C1 = 0).

The cODdition lor two Una to be parallel, whose d.c.'s are I" m" n, and 11, m1, n1 (or proportional d.c.'s are a., bl' c, and a1• hI' cl) is .

I ... ~ m .. m n .. n (or !1.=!!..=!!..)

I .'. I' I 1 .1 b1 c1

8. If I" m" n, and I,. m,. n1 are the actual d.c.'s of two lines thro' the origin. then the d.e.ts of'the lines bisecting the angle between them are proportional to

1.+1. m,+ml n.+D. and 1.-1, m,-m,n.-DI

1 l' l' .2' 1 ' 1

9. Length of Projection. The projection of a segment AD on another line PO is

A'B' ,,; AD cos a

where a is the angle between the lines AD and PQ.

Projection of the join of two points. (0,0,0) and (x, y, z) on a line whose d.c. 's are I, m, n is given by

tx + my + IU. .

Projection of tbe Join or two points (x" Y" Z,).

(Xl' Yl, Z:) on a line whose d.c.'s are I, m, D is given by l(xI - x,) + m(Y.2 - Y,) + n(z, - ZI)'

10. . Important Equations

(I) Eqn. of the pl. II to YZ pl, and at a distance Q from it isx = a i.e; missing co-ordinate (in YZ pl.) = a.

U rheberrec htl lch gesc h utztes Iv! ateri a

(v)

Eqn. of the pi .. II to ZX pI. and at a distance b from it is y '" b or missing co-ordinate= b.

Eqn. of the pI. II to XV pI. and at a distance c from it is z .. c or missing co-ordinate =- c.

The equations of the YZ, ZX, XY planes an: x = 0, y '" 0, Z '" ° respectively .. [Remember, missing co-ordinate = 0).

(if) j(x, y) = 0, where j{x, y) contains only terms.in x and y, represents a cylinder whose generators are II to the axis of missing variable t.e., a-axis here.

Similarly for fly, z) '" 0 and j{z, x) ~ 0 (iii) j(x, y, z) '" 0, represents a surface.

(iv)Equatlons of a curve. j{x, y, z) '" 0, F(z, y, z) = 0 taken together represent the curve of intersection of two surfaces given by . 1(x, y, z) = 0 and F(z, y, z) = o.

(v) . Equations of the eo-ordlnates axes Eqns. of X-ax.is an: y .. 0, Z .. 0 Eqns. of Y-axis are z = 0, x = 0 Eqns. of Z-axis are x ... 0, y ... o.

CHAPTER 3

1. Equations of a plane. in various forms :

(il)

Every equation of first degree In x, y, z represents Q plane.

B.X + by+ c:z + d ," 0 I General form

Any pI. thro' (XI' YI, zl)is

a(x - 1I) + b(y. - Yl) + e(z - %1) ... 0 I One-palm form If I, m, n are the d.c.'s of the normal to the pI. and p, the length of .L from origin 10 the pl., then the pl. is

Ix + my + n1; .. P I Normal/arm

.If the pI. makes intercepts, a, b, c en the axes, then the equation of '

the pl. is .

(iii)

x Y z -+-+-=1 abc

I Intercept form

(v) The pl. passing thro' threepts. (XI' Yl' %1)' (x2• Y1, %2) and (Xl' YI, %1) is

Xl Yl %,

1. The d.e.', of tbe Dormal to the plane are proportional to co-eff. or x, co-efT. or y, co-efT. of Z. in the eqn. of plane ..

X Y Z

XI YI %1

Xl Y2Z1

=0

Three-pt. form

U rheberrec htl lch ge5c h utztes Iv! ateri"

(vI)

3. Angle between two planes whose equations are

al'1" + b,Y + CI% + d,. co 0 and ar + bJl + c.z + dl .. 0 is given by

a ralaz 9 Jr.(albz -:blal)!

cos = ~ ~,sin = ~ r;::-;

V'r.a~ vra: "lEa: vr.a:

e Jr.(a,bl -blal)1 tao :: ....!,_....:.....;:.....:._-=--::..:...._

ralaz

and

4. Condition fat the two planes .

a,x + b,Y + c,z + 41 =- 0 and ar + b.ly + c-j& + d1 = 0 (,) to beperpendicula.r is ala" + bib: + C,CI. = 0

al bl c,

and (iI) to be parallel is ;- = b = ;-

, , I

5. Any planeparaUel to the plane ax + by + cz + d = Ois

ax+by+cz+K"O

Rule to write down the equation of any plane II to a given plane: "In the equation of the given plane. change only the constant term by a new constaru",

6. .Position of tbe origin w.r.t. tbe aagLe between two planes

The quantity ala, + bib, + c,c, is negative or posittve according as the origin lies in the acute or obtuse aogie between the planes a,x + boY + C,% + d, "" 0, ar + b,)' + c.z + d, = 0, where d,. d, are both po5itive.

7. Position of two given points w.r.". a given plane

Rule. Substitute 'he co-ordinates of two poirus in the LH.S. of the equation of the plane (R. H.S. = 0) turn by turn. Then the results are of the same or opposite signs according as the two points are on the same or opposite sides of the plane.

8. Perpendicular distance formula

(I} The .1 distance of the point (x" Y" %,) from the plane fx + my + nz = p (equation in the normal form) is

lx, + my, + 0%, - P.

The complete formula for 1 distance = ± (at + my, + M, - pl.

Rule. Substiuae the co-ordinates of the point in the L.H.S. of the normal equation of the plane after making R.H.S. = O.

(i/) The perpendicular distance of the point (x,. Y" %,) from the plane ax + by + cz + d=!O (equation .in the general form) is

.X, + by, + cz, +d

.JaJ+bJ+cl

U rhcberrec htl lch gesc h utztes M ateri a

(viI)

Rule. Substitue the co-ordinoles of the point of the L.H.S. of the equation of the·plane (with R.H.S. - 0) and .dlvide the result by

J(cQ-eff. of xl2 +(co-eff. of y)2 + (cr>-ejJ. of z)2 .

9. Tb.e planes bisecting the angles between two given planes

QIX + b,Y + cf + dl = 0, art + bzY + cf + dz = 0 are

Qlx+~y+elz+dl ozx+i;y+ ezz +dz ~~==~~=±~r=~~~~

Joi +blz +e~ Jo; +~ +c; ... (1)

Distinction between the two bisec:tina: planes. Write down the equations of the two given planes (with R.H.S. = 0) in which d1, d,. are +ve, then taking +ve sign in (I), we gel that plane which bisects thai angle between two given planes in which the origin lies.

10. .ProJedion on a planc

(0) The projection of a segment AB on a plane 71: is A'B' D AD ees e

where e is the angle between the line AB and the plane 71:.

(b) The projection of a plane area d on a plane 11: is d' >; d cos e

where a is the angle which the plane of areal!. makes with the plane of projection n.

(c) If d., d,.d, are the projections of a plane area d on the co-ordinate planes, the.D

!J. = ~!J.x1 +!J.y +!J.:,.

11. The condition for tbe general bomogen.cou5 equation of second degree in I, y, Z to represent a pair of planes. The condition, for the equation or + bY + cz2 + 2JYz + 2gn- + 2hty = 0 to represent a pair of planes is

a b g

h b f =0

g r c

If the two planes represented by

or + bY + cr + 2JYz + 2gzx + 2irxy = 0, are perpendicular, then a+b+c=O,

i. e., eo-eff. oj xl + co-ejJ. oj y + co-eff. oj r = 0

12. Volume of tbe te.trabedron whose vertices are (XI' YI, %,), (Xl' f:, Zl)' (Xl' fl, Zl) is given by

i.e, abc + Ifgb - ar - bgl - cb1 = 0

X, Yl %,
I X2 f1 %z
y",-
~ x] Yl %J
x. y. z.
!.! rheberrec htl lch gcsc h utztes M ateri" (viii)

CHAPTER 4

t. Equations of a sL line in various forms

(I) a,x + b,Y + ,e,z + d, - 0, a1" + b)y + c?- + d, - 0 I General/arm

(iJ) .. x-x, = y-y, +z-z, (= r) I Distance/arm or Symmetrical/arm

J m n . .

where (x, Y." :.) ,is a point on the line and I, m, n the d.c, 's 'of the

line. " .

(Iii) ~e line thro' (X,1 Y" ,zi) and with ~:c. 's proportional to a, b, c are

X-')(I :::Y-YI ='z-ZI abe Note. Any !JOinl on the line is

(Ir + ""nir'+ Y" nr + :1:,).

(N) ,The line t~f.O' two points (x" Y" l} and (Xl; Y" %,) is

I SYmmetrical/orin

X-X, '=, Y-Y, = z-z" Xl -X, Y, - y, %2 -z, A Hne and a plane:

Let the line be x-x, = y- y, := z-z,

I m n (1)

and the plane be Ill' + by + C% + d '" 0 (2)

(0 The conditions for the line (I) to be II to the plane (2) are [that it should be 1 to. the normal to the plane' and its PQinl (x"y" %,) should , not lie an the plane; Lt.)

al t bm + en - 0 and: .", + by, + czt. + d '-;' 0

, . {ilj The line (I) will be 1 to the plane (2) ifu is II to the no.rmal to. the

I m n

plane. i.e: if - = --, = _;,.

" . . • b C

(iiI) , The line (I) wi'll lie in the plane (2) if [ilis.Ltothe normal to the plane and a paint .(x" Y" %,) an the'line lies an the 'plane i.e: ifl'

al + bm + en .. 0 and 8)[, + 'bYI + el, + d .. 0

Any·plane "through tbe line of intersection of two:planes "(a) Any plane through the line of intersection of two planes a,x + h,y'+ CII + d, a 0, Or + bly + c.;z + dl '" 0

( Equations in general form is .," + b,y + eiz + d, + K(a,x + b~·+ e1z + dJ) = 0

Le; one planc + K (other plane) .. 0 "

(b) . Any plane through a ~i.v~n line

I Thio-point form

/-.

m

n

I Equations in the symmerrica/ form

U rheberrec htl lch gesc h utztes Iv! ateri"

(ix)

is A(x - x!) + 8(y - y!) + C(z - z!) '" 0]

where A.I + 8m + Cn = 0

Cor. Equation a/the plane through one line and II to the other. The

x=x y- y z-z

plane through the line __ I = __ I = __ I and parallel to the line

I! m! n!

~=L=-=- is

12 "'-l ~

!l "'-l nl

Rule. Write down in the two rows the numerators and denominators of the equations of the first line and in the third row, the proportional d.c.ts of the second line. Equate the determinant so formed to zero.

X-XI Y-YI Z-Z,

/, m, n,

= o.

CHAPTER 5

1. Rule to prove the two given lines to be coplanar or intersecting [equations of both lines in the symmetrical form]

Method or plane. Write down the equation of the plane through one line and [I to the other. Show that this plane passes through one point on the other line.

2. Rule to fmd tbe shortest distance (S.D.) between two liDes

X-XI Y-YI z-::I x-x2 Y-Y2 ::-::2

--=--=--,--=--=--

II m] n, 12 m2 n2

(a) [Method or projection] (i) Find the d.c. 's of KL [the shortest distance which is 1 to both the lines. (ll) Then S.D. cs KL = projection

olAe on KL. (iii) S.D. l.e. KL is the line of intersection of the planes AKL (through AB and XL) and eKL (thro' CD and KL).

(b) [Metbod or intersection]. Take any point on line (I) to be K(f.'! + .1'1' m,rl + YI• nt, + ~I) and any point on the second line to be lJ.1{2 + XI' mi2 + Y1, fl{2 + %1)' Find the proportional d.c. 's of KL andfind the values ofr,. r1 by using .thefaclthat KL ..1. to (1) and (2). Putting these values of rl• r2 in K and-L; we find the co-ordinates of K and L. Then by distance formula, we can find the S.D. i.e. KL and the equations olKL are obtained by using the two point/arm.

U rhcberrec htl lch gesc h utztes M ateri a

(x)

(e) [Method of parallel plane]. When one of the lines is in the general form, and the other in the symmetrical form. first find the plane through first line (whose equations are in the general form) and II to the second fine, then the 1. distance of any point on the second line from this plane gives the required S.D.

(d) When both the equations of both the lines are in the general form, fu:st reduce both of them to symmetrical form and proceed as in (a).

3, Equations of two lines ia tbe simplest form

For problems relating to two given lines, take the. Lines to be Y ~ mx, z = c and y = -nIX. z= -c.

4. Any line' intersecting two giveo lines ul = 0, "I ~ 0 and u1 = 0, "l = 0 (equation in the general fonn) is

ul + kivi = 0, u, + k1,,) ~ O.

5. Rule to find the nature of Intersection of three planes

Let the three planes be QIX + bly + clz + d, = 0

a~ + b,Y + c,z + d, = 0 and aiX + b)y + Cl% + dl = 0

(I) Write down the co-effs of these equations thus getting the rectangular array

01 bl ci d, 8 = a. b,. c, dl a) bJ c1 d,

Here 8, is the determinant obtained by omitting the rth column' in 8.

(il) If 8, <F 0, the three planes intersect in a point.

(iiI) If 8, '" 0 and anyone of 81, 8" ~ is non-zero the three planes form a prism.

(iv) If 8. = 0 and 81 '" 8,= 6.J = 0, the three planes intersect in a line.

CHAPTER 6

1. Change the origin. If the origin is shifted to the point if, g, h) then x changes to x + f, Y to Y + g and z to z + h.

2. Change of directions of axes; If I,. m" n, ; 11' m1, 111 ; I" m" III are the d.c, 's of three mutually 1. lines, then

x changes to fr + 'lY + l~ ; Y to mix + m1y + mJ% and z to nr + 112Y + n~.

3. Relations between the d.c.'s of three mutually perpendicular lines. If'l' m" II, ; /1' m2, 111 and '1' ml, n, are the d.c.' of three mutually 1. lines, then

... (A)

11/2 +mlm2 +n11l2 = 0'1

'.1./3 +m.m) +n.n) = 0 ... (5) 12/) +m2m3 +112113 '" 0

U rheberrec htl lch gesc h utztes Iv! ale,;"

(%1)

[Also I,. '2' /, .; m" m~. m1; n" n" n] are the d.c.'s of mutually .Llines, t.e. OX, OY, OZ referred to new lines as axes]

2 2 2 ) .)

I, +/1 + I) '" I I,m, +/2m1 +IJfnJ :. 0

.. mf +m. } +mf = I ... (C) . Ilnl +12f1], +/Jn] = 0 ... (D)

2 2 2

nl +n2 +n)' '" 1 m,n, +mzn2 +m]n] = 0

Cor. If ' .. m .. n, (r =0 1,2,3) are the d.c.'s of three mutually .l (lnes. then

I, m, n,
(a) " I"z n, =±I
I) m, nJ
I, m, n,
(b) and in the determinant 12 ·m, n2
/] mJ ~ eaeh constituent = ± (Us eo-factor)

e.g. I, =± (m,nl - ml'l~) etc.

4. Invariants. If the expression axl + by' + cil + 2fyz + 2gzx + 2hxy be transformed by any change of rectangular axes, then the expressions a + b + C, A + B + C (= ab + be + ca -.r -?! - hl) and D (=abc + 2fgh - a.r - bi' - ehl) remain unaltered.

CHAPTER 7

1. Equations of the sphere

W Equation of the sphere whose centre is origin and radius a is

xl + Y + :t' '" ,; I Standard form

(i/) The sphere whose centre is (a, b. c) and radius r is

(% - a)' + (y - b'f + (z - c)l = r I Central form (iil)Th.e general equation of the sphere is

xl + Y + r + 2ux + 2Yy + 2wz + d = O.

Its centre is (-u, -v, -w)

i. e. [.:.! co-effi. of %,.:.! co-effl. of y, .:.! co-effi. of z ]

2 2 2

and radius = ,luI +v' + w' -d.

(iv) The sphere on the join of (x" Y" z,) and (x" Y" Z,) as diameter is (x - xl> (x - x,) + (y - r,) (y - J,) + (z - z,> (z- tl) = 0

I Diameter form

2. (a) Any sphere through Q' giveD circle

xl + Y +.iI + 2ux+ 2vy + 2wz + d= 0, Ax + By + Cz + D '" 0 is xl +y + Zl + 2m + 2vy + 2wz +d + K (Ax + By + Cz + D) '" 0

U rhcberrec htl lch gesc h "Illes M ateri"

(xii)

i.e: sphere + K(plami) "" O.

(b) Any sphere Ihrouah Ihe elrete of inlersection of two spberes

S,= 0 and S1 ~ 0 is

S, + KS, = 0 or 5, + K(S, - 51) '" 0

3. Rule 10 write down the equation of tangent plane (or plane of contact of polar plane 01) at (x" Y!' .t,) to the given sphere (or conicotd; is, change.

xl to .0"'.' y to »'" r to u,;

1 I 1

x to 2" (% + x,) Y /0 Z (y + Yl) Z to 2' (z + z,)

1 I 1

yz to '2 (yz, + Y,z), zx to '2 (zx, + z,x), . xy ./0 '2 (XJ'1 + x,y).

4. Tangenl plane (line) property. If a plane (line) touches the sphere, then .L distance/rom the centre a/the sphere on the plane (line) must be equal /0 its radius.

5. If two spheres cut orthogonaUy, then the .square of the distance between the centres = sum of the squares of the radii.

The condition of orthogonality of the two spheres

xl + Y + r + 2u,% + 211lY + 2w,% + d, = 0

and :c' + Y + T + 2uzX + 211,Y + 2w;f + d1 "" 0

is 2u,u, + 211,11, + 2w.lI', = d. + d,.

6. Angle of intersection of two ,spheres is given by 'I' +,,' _ d'

cos 9 == 2 where '1' r1 are the radii of the two spheres and d.

r,',

the distance between the centres of the two spheres.

7. Power of a point (XI' y." %.) w.r.t. the sphere S '"' 0 (equation in which co-effs, xl, y, T are unity) is given by 5 •.

8. Length of tangent of the pain I (x" YI' z) from-the sphere S = 0 (the equation where co-effs, of x', y, .r are unity) is -is,.

9. Radical plan.e of two spheres

5, "':c' + Y + r + 2u~ + 211,Y + 2w,z + d, '" 0 5, "';cl + Y + r + 2urt + 2v,y + 2wf" + d, '"' 0

is given by 2(u, - u,)x + 2(v, - vllv + 2(w, - w,)z + d, - d; = 0

i.e. by 51 - 5, = 0,

10. Two spheres in the simplest .form. For problems relating to two given spheres, take their equations 10 be

:c' + Y + z1 + 2u,:c + d = 0, :c' + Y + r + 2u~ + d = 0

.11. Equation of co-axlal system o.f spheres in the simplest form is ;c2+y+zl+2u:c+d=O

where u is the parameter.

The limiting points of this co-axial system are (± -id, 0, 0).

U rhcberrec htl lch gesc h utztes M ateri"

(xiii')

CHAP1'ER8

General equation of tbe cone wbose vertex is tbe orilla is aJ? + bY + cz1 + 2.6': + 2grx + 2hxy = O.

Note. The de. '.1' oflhe ge.ne.raJOI' of the cone (with vertex at origin) sallsfy 1M equation of 1M cone.

The equation of quadratic (second degree) cone througb the axes is .6':+gzx+hxy=O Equation of 8 ri&ht circular cone. The equation of the right circuler cone wbose vertex is origin; axis, the z-axls; and the semi-vertical angle a, is

xl + Y = z2 tan2 a I Standard form

Equation of the enveloping cone from the point (x" Y" z,) to a sphere (or a conicoid) is SS, = TZ

where S = 0 is the equation of the sphere (or conicoid) S, = the result obtained on putting (x,. Y" ZI) in S

and T = 0 is the equation of tangent plane at (xl' Y,. %,).

Rule to prove that a given numerical equation represents a cone and to find Its vertex.

(I) Make the given equation homogeneous in x, Y,i!. t by introducing proper powers of t and the R.H.S. = O. Let it be denoted by F(x, Y, z, I) = O.

of ilF of of of

(il) Find Ox' Oy' Oz'at and put f = I and put ax = 0

aF = 0, aF :< O. aF = 0

Oy {)z al

(iiI) Solve for x, Y. Z any three of the above four equations.

(ill) Put these values of x, y, z in the fourth equation. If the equation is satisfied, then the given equation represents a cone whose vertex is given by values of x, y, z found in step (ii/).

6. Tangent plane at (x"y" z,) to tbe cone

or + by + czl + 2.6': + 2gzx + 2hxy = 0 is

an, + b.Y.v, + CUI + ftyz, + Y,z) + g(zx, + z,x) + h(xy, + x,Y) = 0 Notations:

1.

2.

3.

4.

5.

a h g

(I) 0 = h b f = abc + 2fgh - afZ - bgZ - elr. g f c

(iI) A, D, C, F, G. H are the co-factors of a. b, c. f, g, h in a h g

0= h b f ,

s f c

urhcbcrre<;htlich gcschillztcs Material

(xiv)

i.e. A ~ be - f', B '=' co - gz, C= ab -Ir; f = gh - af. G = hi - bg, H = fg - ch.

(ii') BC - f1 = 00, CA - G' '" bO,

GH - AF = /D. HF - 80 '" gO,

AB-Hl=cO FG- CH '"" hO.

(/v)

a h g u h b f v g few U \I W 0

'" --(AuI + BvI + Cw' + 2Fvw + 2Gwu + 2Huv)

where A, B. C, F. G, H are the co-factors of a. b. c etc. in the

determinant O.

7. The condition or tangency of the plane lx + my + nz '" 0 and the cone ar + bY + cz1 + 2Jj>r + 2gu + 2hxy = 0

is AP + 8m2 + or + 2Fmn + 2GnI +2H/m = 0

8. Equation of reciprocal cone of the cone

aX' + by + cz1 + 2JY: + 2gz;f + 2hzy .. 0

is .A:!? + By + Cz2 + 2Fyz + 20U' + 2Hxy "" 0

where A, B. C, E, G, H are the co-factors of 0, b, c. J, g; h respectively

o h g

in the determinant h b f

g f c.

9_ Condition for the cone or + by + cz1 + 2M + 2gzx + 'lhxy = 0 (/) to have three mutually 1. generators is a + b + c = 0

{i.e. sum oflXrf'j/s. of.r, Y. r = 0]

(II) to have three mutually 1. tangent planes is A+8+C=O

i.e. ab + be + co - r - gl- Ir '" 0

CHAPTER to (CoDicold)

I. Equations or conkold in the simplest form (Standard forms)

xl l z2

-+-+-=1

02 b2 c2

(a) Ellipsoid

Xl y2 Z2

(b) Hyperboloid of one sheet -+---:= I 02 . b' c2

. Xl yI Z2

(c) H"perbolold or /Wo sheets - - - - - = I

-" 'J 02 b' c'

(d) Central con/cold tW + by + cz1 = I

U rhcberrec htl lch ge5c h utztes M ateri"

(xv)

[Note. Equation of the central conicoid, when not stated, should be lakcn as ax' + by + ex' = I.)

2. TBng~nt plane at (x,. Y" z,> to the conicoid ar + bY + cil"" 1 is au, + h.YY, + 02", = 1.

3. Condition of tangency of B. plane and. central conlcoid.

The condition that the plane Ix + my + nz .. p should touch the conicoid ar+bY+cil= I. is

e m2 n2 -+-. -+-=/ abc

Cor: Any tangent plane to the central c:onicoid ·ar + bY +d "" I

. /2 m1 n1

is Ix+my+nz= -+-+-.

abc

4. Equation of the dlreetor splle.1'e of the central conicoid is

I 1 1

xl + Y + z2 '" -+-+-. abc

5. Equations of tbe normal at (x" Y,. %,) to the ellipsoid are

x-x, =y-y, = Z-%,

x,la' y,/b' z,lc'

t.e:

where F(x, y, z) = 0 is the equation of the ellipsoid. (or all)' con/coiti) EquBlions of tbe normal al (x,. Y" z,) of the ellipsoid. in tbe actual direction c.oslnes form, are

x-x, = Y-YI = Z-ZI . F, 102 PY, I bl F, I c'

wherep is the length of 1. from the centre of the ellipsoid on the tangent plane at (x,. Y" %,).

6. Equation of polar plane and the plane of contact of (x,. Y" z,> w.r.t. the central conicoid ar + by + d = 1 is the same as the equation a/the langent plane at (x)' Y" z,) Le.

au, + b.Y.Y, + cu, = I

7. Equation or enveJopine: cone from the point (xl' Y" z,) to the central conicoid S c 0 is given by SS,= Tl.

8. Equatlo·n or tbe plane of the seetlon oCthe central conicoid whose centre is the given point (x" Y" z,) is given by T '" S,.

U rheberrec htl lch ge5c h utztes Iv! ateri a

(XVI)

THE CONE

I. Equation of the cone in the simplest form is

a.r + by + cil .. 0 I Standard form

2. Tangent plane at (x" Y" z,) to the cone a.r + by + czl = 0 is

au, + byy, + CUI ~ O.

3. Condition of tangency. The condition that the plane

Ix + my + nz '" 0 may touch the cone a.r + bY + al '" 0 is that

l' m1 n1

-+-+-=0.

abc

4. The polar plane or the plane of contact of the point (x,. Y" z) w.r. f the cone is given by

atX] + h»', + =, == 0

5. The equation of plane seeuen of the cone whose centre is at the point (x" Y" %,) is given by T '" 8,.

THt: PARABOLOID

I. Equation of tbe paraboloid in the simplest fonn is

ax' + by = q. I Standard form

2. Tangent plane at (x" Y" ;,) is on, + byy, = z + %,.

3. Condition or tangency. The condition for the plane

Ix + my + nz = p to touch the paraboloid ax;l + bY .. 2=,. is

[2 m1

~+_ .. - = -2np. a b

4. Equation of Ihepolar plane or the phlne 01 contact of point (x,. Y" z,) w.r.t the paraboloid is au, + h»', == Z + %, [This is same as the equation of longenJ plane at (x,. Y" %,»). .

5. The enveloping cone from the point (:c" Yl' I,) to paraboloid S = 0 is given by SS, = Tl.

6. Equa.lions of normal to an eUiptic paraboloid

x2 y2

-+-:; 2z at me point (:cl• Y" z,) are oj bl

x-x Y-Y z=z

-- .. -~ :; --~ =--'

x,la Y,/h -I

or

where F (x, y, z) '" 0 is the equation of paraboloid

U rhcberrec htl lch gesc h utztes M ate,; a

(xviI)

S" GENERAL METHODS IV.lmp.1

[We give below some general me/hods which are applicable to sphere-cone, cylinder and all the conicotd The student is .strongly advised to practise these after doing the chapter on sphere.]

Art. 1. Find the equation of tangent plane at (x" y.,. x,) 10 the following

YUtjoces :

(a) Sphere x:' +y + r = al

(b) Sphel'er +y + zl + 2ux + 2vy + 2m + d = 0 (e) Cone ar· + by' + cr + .2JYz + 2gtx + 2hxy = 0 (d) Cylinder ar + by + 2hxy + 28% + 2JY + C'" 0 (e) The central conicoid or + by + cT = I

xl. y' z' -+-+-=1 a' 6'· c'

(f) Ellipsoid

x2 y'.

(g) Paraboloid - + - = 2:. a' h'

SoL As an illustration, we solve the (g) part The equation of the paraboloid is

x' y'

-+-=·2z

a' b'

Any tangent line through. (x,. Y" Z,) is

... (1)

X-x, = Y- Y, = Z-Z,

m n

Any point on this line is (/1' + x" mr + Y" 111' + :Z,) If it lies on paraboloid (1), then

(/r+x,)' (mr+y,)' -2( ~)

, + , - nr+_,

o b

... (2)

or

r,(f_+ m')+2r(bcl + my I _n)+(X~ +y~ -2ZI)=0

02 b' a' bI a' b'

But since (x" Y" :,) lies on the paraboloid (I), x' y2

.. -L+-'-=2z

a' b' ,

Using (4), (3) reduces to

... (3)

... (4)

2( /2 ml) (IX, my I ) 0

r .--'-+- +21' -+-, -n =

a' b' a' b-

. . One value of r in (5) "" 0

... (5)

U rhcberrec htl lch gesc h utztes M ateri"

BUI since the line (2) touches the paraboloid (I). .. line (2) meets (I) in two conicident points.

. . both values of r must be equal. But since one value of r is zero :. the other value of r is also zero.

.. co-effs. of r in (5) is zero.

lx, my I

-+--71='0 ('6)

a2 . b, ...

To find the equation of tangent plane at (x" Yl' =,>

t. e. 10 find locus of tangent lines (2), we have 10 eliminate I, m, n from (2) and (6).

Eliminating I, m, n from (2) and (6). we get,

(X-XI);~ +(Y-YI):~ -(:-z])=o

XXI xr .»'i Y; ---+----:+z] =0

a2 a2 b1 b1

XX, »'1 x: ,Y:

-+--:=-+--z· a! bl a2 b1 I

Adding - ZI to both sides,

XXI YYJ xl . yf

-. -+--(:+ZI)=-+--2z] ",0

a2 b1 02 b1

(xv;;,)

i.e.

or

or

I, Using (4)

or ~' + Y.Vbl' - (z + Z,) = 0 which is the required equation of tangent plane

o '

at (x" Yl' =,) 10 paraboloid (1).

Please try tbe otber parts yourself exactly as above.

Rule to write down tbe equation of taneent plane at (x,. F,. :,,). In the' given equation of the surface, change

I ,

% 10 '2 (% +Z,),

I Rememberl To find the condition thaI the plane lx + my + nz '" p may toucb the following surfaces ;

1

yz 10 '2 (yz, + YIZ),

and

1

x 10-(X+X,), 2

Art. 2.

(a) :.r? + Y + r = a' (e) a:.r? + by + cz2 = I

U rhcberrec htl lch gesc h "Illes M ateri"

(xix)

(e)

[Method of plane of eontaet]

As an illustration we do (d) part. The given surface is or + uy= 2z

and the plane is ix + my + nz '" p

Let the plane touch the surface (I) at (x" Y" Z,) ... Equation of tangent plane at (x" Y" zt> to (I) is

a.tt', + bY.v! = t + 1, ... (3) I Rule of tangent plane

=, + bY.v] - : = I"

Since this is the same as the plane (2), :. Comparing (2) and (3), we have

... (1) ... (2)

or

4X, = by, =.:.!. = 5..

m n p

... .. -I -m .-p

which gives, x, =-,y, ='-b ,Z, =-.

na n n

Since (x,. Y" z,) lie5 on (1).

:. a(:~r +b(:;J =2(':)

or

/1 ml .;_+- = "':2np a h

which is the required condition.

The point of contact is (X,. y,. :,)

( _ s; _!!!.... _ E.).

na nb n

or

Please try yourselr the other parts as above.

[Ans. (a) dI(P + ",1 + fil) .. ,r (b) d'P + /i'm' - c'rr = ,r

12 . m2 . n2 2

(c) -;+b+~= p (d) diP - clfil ~ -2mp]

Art. 3. Find the locus of the points oj contact oj the tangent planes to (he following surfaces which pass through the point (x,, Y,. Z,),

(a) xl + Y + z' .. a' (b) x' + y + z' + 2ux + 2~ + 2wz + d" 0

Xl yl Zl

(e) or -I- by + CZl = 1 (d) 7-bF-cr=1

(e) or + by '" 2cz

Sol. As an illustration we do the (b) pan

The equation of the sphere is

xl + l' + z' + 2ux + 2~ + 2m + d "" 0

... ( I)

U rheberrec htl lch ge5c h utztes Iv! ateri a

(xx)

Let (x', y', z') be the point of contact of any tangent plane to the surface

(I).

:. Equation of tangent plane at (x'. v', z'] to (I) is

xx' + )')I' + zz' + u{x + x') + v(y + y') + w(z + r) + d = 0 Since it passes through (x" y" z.),

:. x?" + Y,Y' + Z,Z' + u(x, +x') + V(Y, + y) + w(Z, + z') + d = 0

:. Locus of the points of contact (x', y'; z'), [change (x', y', l') to (r, y, l)] is

xx, + )')I, + zz, + u{x + x,) + v(y + y,) + w(z + z,) + d = O.

This plane is called the plane of contact of the point (x" Y" z,) w.r.t the given surface (1).

Note. The equation of the plane of contact of (x" Y,. t,) is the same as the equation 0/ 'he rangen' plane at (x.,. Y" :,).

Please try otber parts yourself.

Art. 4. Find the equation of the polar plane 0/(.1',. Y,• zt> 1II.r.t the following surfaces:

(a) . .r + t + il '" tt (b) .r + y + z2 + 2UX' + 2ry + 2wz + d = 0

(e) t:W + by + czl = 1

Xl yl

(e) -+-=2:

aJ b'

(f) t:W + by + cz2 + 2M + 2,gn + 2hry = O. Sol. As an illustration. we do the (e) part. The equation of the surface is

.1'2 y'

-r+1 = 2z ... (1)

a . b

Let P be the point (x" Y,• Z,) QR any chord through P which meets the surface (1) in Q and R. Let S(x. y. z) be the harmonic conjugate of P 111. r. 1. Q and R.

Then llK:uS or 5 is called the polar plane or p.

The point which divides PS in the mtio Ir. : I.

(.b:+.1" ky+y, b+Z,)

~''''k+I'k+1

If it lies on thesurface (I). then

...!...(kx+X')' l +_!_(ky+ y,)l = .,( h+ZI.)

oj k + 1 bl k + I 1. k + I '

U rheberrec htl lch gesc h utztes M ateri"

or

or

which is a quadratlc in k .

. ,. PS is divided hanitonically, i:e: internally and externally in the same ratio at Q and R, :. the quadratic (2) has equal and opposite roots .

. '. sum of roots = 0 :, the co-eff of k = O.

i.e.,

or xx, + .»', = z+ Z

0.1 b2 I

which is the required equation of the polar plane,

[Remember: The equation of the polar plane of (x,.' Y" I,) is the same as the equation of tangent plane at (x" y,. I,)].

Please try tbe otber parts yourself as above.

Art. 5. Find the pole of the plane lx + my + lIZ = P w.r.t. the following surfaces :

(d) or + by = 2z

Sol. As an illustration, we do the (e) pari.

x2 y2 Z2

The given surface is 2"+2+2"= I abc

and the plane is lx + my + nz ~ p

Let (x" Y" I,) be the pole of (2) w.r.t. (I) Then the polar plane of (X,. y" II) w.r.t. (1) is

xx, + .»'1 + ZZ, = 1

a1 b'2 cl

Since it is the same as (2) : .. comparing we have

x, 10.2 = y, I bl. = 2:, I c1 ==..!..

I m n p

0.21 b2m c2n

•• XI =-'YI =--'Zl =-

P p' P

... {I) ... (2)

U rheberrcc htl lch gesc h utztes M ateri a

(:0:;')

( aZI b1m c2n) Hence the pole is (x" Y" z,) or p'p'p

Please try the other parts yourselr as above.

(b) (_!_,.!!!.. . ..!!...)

ap bp cp

(d) (-_!_,- .!!!...,- p..)

no nb n

Art. 6. To find the equation of the polar of tbe line

X-Xl Y- Yl ,1:-:1

-,- '" -- =0 -. - w. r.t the following swjaces .'

m n

(a') ax' + by + 2z

As an illusmuion, we do the (e) part.

. .. x-x, Y- y, Z-Z,

The given hne IS -- = -- = -- ... (1)

I m n

x2 yl

and the surface is -+-=2z ... (2)

Ol bl

Any point on the line (I) is (Ir + x" mr + y" nr + z,). Equation of the polar plane of this point w.r.1 the surface (2) is

x(/r+x,) y(mr+ y,) . .

__;_-'2 --'--'- + 2 = Z + (nr +z,)

a b

or XXI + WI -z-zl +r (..!=_+ ym -n) = 0

02 b2 02 b2

which, for all values of r passes through the line

XXI+.m.. -Z-ZI =o,(..!=_+ my)-n::o

02 b2 02 b2

.. Required equations of the polar line are

.l".X, + W, ~+!!l[:: n

7 b2'=z+z" a1 b1 .•

Please try the other parts yourself as above.

I Rule of tangent plane

..

U rhcberrec htl lch gesc h utztes M ateri"

(xxiii)

Art 7. To find the equaJion of'the plane section o/the/ollowing surfaces whose centre is (XI' YI, ZI)'

(a) xl + Y + it = a2 (b) ax2 + bY + r:r = 1

(d) xl + Y + r + 2ux + 2\')' + 2wz + d = 0

(e) ax2 + bY + cr + 2fyz + 2gu + 21ay = O. As an illustration, we do the (d) part.

The given surface is.r + y + it + 2ux + 2vy + 2wz + d = 0 ... (1) Let (XI' Y., :1) be the mid-point of any chord.

lei the equations of the chord through (XI' YI, :1) be

or

X-XI = Y-YI = %-ZI

I m n

Any point on this line is Ir + xI' mr + YI, nrl +' %1

If it lies on (I), then.

(lr + X.)l + (m,. + Y.I' + (llr. + %)1 + 211 (Ir + Xl) + 2v (m,. + Y.)

+ 2w (nr + z) + d = 0 reP + m1+ nI) + 2,. (lx, + my. + f1Z. + uJ + vm + wn)

+ (XII + Y12+ Z.· + 2uxi + 2\Yl + 2wzI + d) = 0

... (3)

... (2)

which is a quadratic in r.

Since (xl' YI, z.) is the mid-point of the chord .: the quadratic (3) has equal and opposite roots, i.e., sum of the roots = O.

., co-effs of r '" 0

or lx, + my. + f1Z. + III + mv + IIW = 0

or I(x. + 1/) + m (v. + v) + II (z. + lV) = 0 ... (4) :. locus of chords (2) [eliminating I, m, II from (2) and (4)] is

(x - x.) (XI + II) + (v - Y.) (v. + v) + (z - %1) (ZI + w) = 0

or XXI + W. + %%. + Ul' + II)' + wz= X.I + y/+ z/ + ux. + "'. + wz.

which is the required equation of the plane of the section.

Please try the other parts yourself as above.

(Remember: The equation of the plane of the section of a given surfaces S = 0 whose centre is (Xl' y., z.> is given by T = s •. where T = 0 is the tangent plane at (XI' y., %1)).

Art. 8. To find the equation 0/ enveloping cone (0,. tangent cone)/rom the point (x.' Y., z.) to the following surfaces :

(a) xl + Y + r = a2 (b) ax' + by + r:r = 1

~l yl Z2

(c) ---+-=1 01 b1 c1

(d) ax2 + bY = 2cz .

IV- Imp)

urheberre<;htlich geschutztes Malerial

(xxiv)

SoL As an illustration we do the (c) part The given surface is

x2 yl Zl

---+--1 (1)

a2 b2 c2 - •••

Let P (x, y, z) be the given point

Let Q (x, y, z) be any point on a tangent from P to the surface (I), The point which divides PQ in the ratio k: 1 is

(.b:+x" Ay+y" kz+Z,)

k+1 k+1 k+1

If it lies on the surface, (1), then

or

or

... (2)

which is a quadratic in k. .

Since PQ touches the surface (I). r. the quadratic (2) has equal roots, for which discriminant 'IT - 4ac' of (2) is zero.

4 (xx' _ W, + ZZ, _1)2

.. ,al bl cl

(X2 y2 Zl )(X2 y2 Z2 ) (. .)2

or ....,... -- +--1 ....L _....L+....L_I =,a, _ W, + zz, -I

a) b2 c2 a2 bl r? 02 b2 c2

which is of the form 55, .. T1 (Remember(

where S ~ 0 is the given surface, 5, is the expression obtained on putting (x"y" z,), the given point, in S and T = 0 is the equation of the tangent plane at (x" y,. z,> to S = o.

Please try Ibe otberparts yourself.

U rhcberrec htl lch gesc h utztes M ateri"

Art 1. Definitions

1

Co ... ordinates

Origin. Lei X'OX, Y'OY and 2'02 be three mutually perpendicular

straight lines in space, intersecting at O. z

Then the point 0 is called the origin.

Axes. The three fixed straight lines X'OX, Y'OY and Z'02 are respectively called X-axis; Ysaxis and Z-axis. The three lines taken together are called rectangular co-ordinate axes. -

Co-ordinate planes. The plane containing the axes of Y and 2 is called the Yl-plane. Thus plane VOl is the YZplane. Similarly, th.e plane ZOX containing

z

.• y'

·············h---_

X' .0 X

y z:

Fig. 1

C J-------,r--.,. M

y'

y



A X

N

Fig. 2

the axes of l and X is called the ZX-plane and the plane XOY is called the XY-plane and contains X-ax.is and Y-axis.

The above three planes are together called the rectangular co-'Ordinate plane or simply co-ordinate planes.

Oetants, The three co-ordinate planes divide the whole space into 8 parts called the octants.

Ce-ordinates of It point. Lei P be any point in space. Draw, through P, three planes parallel to the three co-ordinate planes meeting the axes of X, Y and Z in the points A, Band C respectively. Then if

OA = x, 08 eo y and DC = z,

U rhcberrec htl lch gesc h utztes M ate,i a

2

Golden Solid Geometry

M

y

x

L --+-_;.;_--( PIx, y, z)

y

a

x

the three numbers x, y, z taken in this order are called the co-ordinates of the point P and we refer the point as (x, y, z) or P(x, )" s), Anyone of these x, y, z will be poshive or negative according as it is measured from 0 along the corresponding axis, in the positive or negative direction.

Some other ways of defining the co-ordinates of 8. point. We have seen that In order to determine co-ordinates of a point P, we have to draw three planes through P respectively parallel to co-ordinate planes. The three planes through P and the three co-ordinate planes form a parallelopiped, which has six rectangular faces consisting of three pairs of parallel planes viz.

PMAN, LeOS; PNBL, MAoe; PLCM, NBOA

(I) Then we have, .Z

x= OA = CM = LP = 1. from P on

YZ plane.

y = OB = AN = MP = 1. from P on ZX plane.

z '" OC '" AM '" NP = 1. from P on XY plane.

Thus the co-ordinates x, y. z of a point P, are the perpcndicular distances of P from the three co-or-diaate planes YZ. ZX and XV res-pectively. (C.T.M.)

(;1) Since in Fig. 2, OA 1. plane PMAN, therefore OA 1. 10 the line PA which lies in the planePMAN. [Because a line 1. 10 a plane is 1. to every line in the plane].

PA 1. OA

N

Rg .. 3

Similarly PB J. 08 and rc 1. OC.

Thus the co-ordinates x, y. 'Z of a pOint Pare also the distances from the o.tigln 0 of the feet A. D, C or the perpendiculars from the poiot

P to the co-ordinate axes X'OX, Y'OV, Z·OZ respectively. (C.T.M.)

(iiI) We have, [see Fig. (2)] z P(x. Y. z)

NP = AM .. OC '" z +

AN'" OB '" Y

and

OA"'x

Thus, if we draw the line PN perpendicular to the XV plane meeting it at N and line NA parallel to the line OY meeting OX at A, we have

OA = x, AN = y aod NP "'z,. Important Note. On the YZ·planc, u point has ;r co-ordinate as zero and similarly

y

Fig. 4

x

:r. :i N

U rhcberrec htl lch gC5C h utztes M ateri"

Co-ordinates

3

on the ZX-plane and XY-plane, the y and z co-ordlnates of a point vanish. Thus in Fig. (2) we have the co-ordinates of L as to, y, z), M as (I, 0, z) and .N as (x, y, 0). Moreover, the co-ordinates of 0 are (0, 0, 0).

Art. 2. Distanc.c between two points

To lind the distance between two points (XL' Y" Zl) and (x" Y" z). Let P (x,. Y" ;:,) and Q (Xl' Y2' ;:,) be two given points.Through P and Q draw PL and QM .Ls to the XV-plane meeting it in the points Land M respectively.

Then in the XY·plane L is thepoint (xl' y,) and M is (x" y), so that LM' = (x, - x,Y + (v, - Y,)'

... (1) Now, through P, draw PR L OM. Then clearly PR = LM

and OR '"' QM - RM

'" QM - PL '" %, - %,

Then in the right angled triangle PQR,

PQl = PR' + QR' [By Pythagoras Theorem]

'" LM' + OR' Y

= (Xl - X,)! + (vI - Y,Y + (%, - t,)'

I 2 1 1

.. PQ= ~(Xl-xtl +(y -y,) +(z,;-ZJ)

z

o

x

L .......••. M

reT.M·1

Thus

Distance between two points

_ r(dlrrerenccoh:co-ordiDBtes)l ]

Vl +( diff. of yco-ordinates)2 + (diff. ofz co-ordinates)l

IC.T.M·I

Note. The result is similar 10 the corresponding formula for the distance between

two points in plane co-ordinate geometry.

Cor. Distance of any point (x, y, z) from the Origin

= J(x - 0)2 +{y - 0)2 +{z- 0)2 == ~Xl + yl +zl

Example I. Find the distance between twa points' (4, 3, -6) and (-2. I, -3).

Sol. The given points are (4, 3, -6) and (-2, .1, -3)

... required distance "" ~(4+2f +(3_1)2 +{-fj+3)2 ../36+4 +9 = .f49 := 7

U rhcberrec htl lch gesc h utztes M ateri a

4

Golden Solid Geometry

Method to prove by distances that tbethree points A, H,C arecolUnear: (I) Find the three distances AB, BC and CA.

(2) 'Then if the sum of any two distances is equal /0 the third. the Ihree given points are collinear.

ABC AB + BC = AC

A C B

AC+ CB == AB

. Example 2. Prove by distances that the three points. (a) (-2, J, 5), (I, 2, J) and (7, 0, -1) are collinear. (b) (3, 2, -4), (5, 4, -6) and (9, 8, -10) are collinear.

Sol. (a) Let A(-2, 3, 5), B(I, 2, 3) and C(7, 0, -1) denote the given points. Then

AB == J(1+2)2 +(2-3)2 +(3_5)2 "= ./9+1+4 = JI4 Be= J(7 - 1)2 +(0 - 2)2 + (-J _3)2

..JJ6 +4 + 16 = JS6 = 2..fli

AC '" J(7+2t+(0-3)1 +(_1_5)2

./81+9+36 ==./l26 = 3M.

Since in this case, AS + BC ... AC

r : •. A, B, C are collinear, which proves the result. (b) Please try yourself.

Example 3. Show tha: the points (0, 7, 10), (-J, 6, 6) and (-4, 9, 6) form an Isosceles righl-angled triangle.

Sol. Let A(O, 7, 10), BH, 6, 6) and C(-4, 9,6) denote the given points.

Then AD = JH-O)2 + (6_7)2 +(6-10)2 = ./1 + I + 16 = 312

and

BC = A-4+1)1+(9_6)1+(6_6)2 =J9"+9=J.fi AC = J(..4-0)2 +(9 _7)2 +(6-1.0)2 == ./16+4 + 16 == 6

Since AB == BC, :. the triangle ABC is isosceles.

Further AB' +: BCI '" (3.J2)' + (3"2)1

== 18 + 18 = 36

Also AC2 = 36·

AB' + Be' = ACI, :. .1ABC is a rt.. angle.

Hence the three given points A, B, C form an isosceles rt, angled triangle. Worldng Rule (or problems on detection or the nature of rour sided

geometrical figures : Ie. T.M.I

U rheberrec htl lch ge5c h utztes Iv! ateri"

Co-orriina18s

5

For proving a four-sided figure to be

(a) Parallelolram, show that (/) opposite sides are equal and (iI) diagonals are unequal.

(b) Rhombus, show that (I) four sides are equa! and (il) the diagonals are unequal.

(c) Square, show that (I) four sides are equal and (iI) the diagonals are equal.

(d) Rectangle, show that (i) opposite sides are equal and (it) the

d/agonals are equal. .

Example 4. Show that the points (I. 2, 3), (-I, -2, -I), (2, 3, 2) and (4, 7, 6) are the vertices of a parallelogram.

Sol. The given points are A(l, 2, 3), 8(-1, -2, -I), C(2, 3, 2) and D( 4, 7, 6). Then, by distance fonnula,

AB J(-I-I)2 +(_2_2)2 +(-I-N ./4+16+16=6.

/D(4,7.6) 7(""')

AL..(-,,-2,-3-'-------:!B(-', -2, -1)

8C '" J(2 + 1)2 +(3+2)2 +(2 + 1)2 = ./9 + 25+ 9 =.[.fj

CD J(4-2)2 +(7_3)2 +(6-2)2 .J4+16+16 =6

and

DA

'" .J9+25+9 = Jij

Again AC "" J(2-1)2 +(3_2)1 +(2_3)2

.J1+1+1=./3

and

8D J(4 + 1)2 +(7+2)2 +(6+1)2

'" ..J25+81+49;"'..,ti5S

U rhcberrec htl lch gesc h utztes M ateri a

6

Golden Solid Geometry

Now since AB =- CD and BC = DA, i.e .• opposite sides are equal and AC ~ BD. i.e., the diagonals are not equal.

:. A, B, C, D represent the vertices of a parallelogram.

Example 5. Show that the points (1, 3. 4), (-I, 6, 10). (-7. 4, 7), (-5, J, I) are the vertices of a rhombus.

Sol. Please try yourself.

[Hint. If A, S, C, 0 denote the given points in the given order, show that AB=BO-CD-DA i.e., the sides arc equal and AC '# BO i.e., the diagonals arc not equal.]

Example 6. (a) A. B are points (1, 3, 4) and (-1. 5, -4) and P, Q . variable point. Find the locus of P such that

PAl - PBl '" 2c1.

(b) Find the locus of Q point which Is equidistant from two given points (3, 4, -5) and (-2, 1, 4).

Sol. (a) Let P be the point (x, y, z), .

Since we have, .

PN - PBl '" 2c1.

:. (x - 2)' + (y - 3)1 + (z - 4f - [(x + 2)' + (y - 5)' + (z + 4)1 = 2c1

or r- 4x + 4 + Y - 6y + 9 + r- 8z + 16

-{r+ 4x + 4 + y- lOy + 2S + r+ Hz + 16) = 2c1

or -ax + 4y - 16z - 16 = 2c'

or 4x-2y+8z+8+c1=0

which is the required equation of locus.

(b) Please try yourself. [ADs. lOx + 6y - 18z - 29 '" 0] [HInt. Given PA=PB where P is the variable point and A. B arc the given points (3, 4, -5) and (-2, I, 4)]

Example 7. Find the locus of a point which moves so that the sum of its distance from the poims (a, 0, 0) and (-0. 0, 0) is constant (- 1k).

Sol. Let P(x, y,. z) be any point on the locus and let A(a, 0, 0) and B(-o, 0, 0) be the given points.

Then we are given that PA + PB = 2k

J(x-a)2 + y2 +Z2 +J(x+a)2 + y2 +z2 = 2k

or

or

J(x-a)2 + y2 +z1 = 2k -J(x+Q)2 + i +z2 Squaring both sides. we have

(x _a)2 + y2 +Z2 = 4k2 +(x+a)2 + l +Z2 -4kJ(x+af + l +Z2

or

4.t2 + (x + a)2 - (x - of 4.t2 + 4ax

Jrhcbcrre<;htliGh Qcschiltztcs Materia

Co-ordinates

or

or

or

7

x2 yI +Z2

or -+--- '" 1

k2 k2 _02

which is the required locus.

Example 8. Find the locus of a point whose distance from the x-axis

ls twice its distanoe from the yz.plane. (Agra 1981)

SoL Let the moving point be

Pea, p, y). Then

Its distance from x.axis

= Jfj2 +y2

and its distance from theje-plane = a According to the question,

Jp2 +y2 =- 2a Squaring, pI + f '" 4al

:. Locus of P (a, p, y) is

Y + zl = 4.%'.

Art. 3. SecUon formula

To nud the oo--ordlnates of a point wblch divides the join of the points (XI' )/t' <:1)' (~. '1' tJ internally In the ratio mimi'

(Milhila /982; Meerut 1986)

Let R (x. y, %) divide the join of given points P(xl, YI' %1) and Q(x1, Y2, %2) in the ratio "'1 : "'J'

. Draw PL, RN and QM .Ls on the XV plane,

The lines PL, QM and RN clearly lie in one plane (',' as PL, QM, RN are .is to the same plane XV, r. they are II. Further these II lines are cut by the same line PRQ. :. they are coplanar). Thus L, N, M,

z

Q(x,. y,. zJ

,.' B

(x, y, z) ~ ... ," " 'R' p: m, '

(x" y" z,)

o

x

v

U rhcberrec htl lch gesc h utztes M ateri"

8

Golden Solid Geometry

the feet of Ls lie in a st. line. the line of intersection of this plane and the

pl .. XY. .

Through R, draw a st, line ARB II LNM, to meet LP (produced) in A

and MQ in B.

Now in as APR and BRQ, we have L ARP = L QRB

L PAR ==- L RBQ

L APR = L RQB

The 6s APR and BRQ are similar, and so

PA ==- PR =~

'BQ RQ m2

Now PA '" AL - LP '" RN - LP = z - z, BQ= MQ - MB = MQ - RN = Zl - Z

(vertically opp. Ls)

(alternate angles)

... (1)

... From (I),

Z-ZI ~

zl-z m2

m~ - m~1 = mlz1- mlz (m, + ml)z = mlzl + m~.

mlzZ+ mtzi

z-

or or

or

Similarly,

x ..

ml.ll +m1x, ml.+m%

and

Y = ml +ml

Hence the co-ordinates of the pO/III which divides thejoin of (x.' Yl' :-.) and (Xl' YI, Zl) in the ratio m1 : III] are

('mIXl + "'2x• , m.y} + m2Y. , m.zl + mlz, .)

~+~ ~+~ ~+~.

Rule to write down the CfHIrdinate of the point whicb divides the join of two given points internally in tbe ratio mt : m1,

(I) Multiply m. by the x-co-ordmate of the point remote/rom m, and ml by the x-co-ordinate of the point remote from m1 as shown by the arrows.

(2) Add these products. and

(3) divide the sum by ml + m. .

This gives the x-co-ordinate 0/ the point of division. Similarly for the y and z co-ordinates.

(X" Y"z,)

U rheberrec htl lch gesc h "Illes M aleri a

Co-ordinates

9

Cor. 1. The co-ordinates of the mid point of the join of (x" YI' Z,> and (x2' Yl' Zl) are

IC.T.M·I

[ sum of X co-ordinates sum of yeo-ordinates 2 ' 2 '

sum of Z C~rdinates ]

Proof. Let P, Q be the points (xl' Y" Z,) and (Xl' YI, Z2) and M(x, y, z) be the required mid. point.

PM 1

Then PM=MQ or MQ =1 ~

i.e;

I.X2 + l.XI XI + X2

:. x'" 1+1 :-2-

p

(~" Y" Z,)

M

a (x" y ... zJ

, . YI+Y2

Smularly y= -2-'

Z, +z2 z=--

2

Cor. 2. The co-ordinates of the point which divides two points (x" Y" Z,), (xl' Yl' Z1) externally in the rauo ml : m1 are obtained by finding the co-ordinates of the point which divides the join of given points internally in the ratio of m, : -~, i.e., change the sign of one of the ratios and proceed as in the case of internal division. They are :

.{mIX2 -IIIJXt , mtYl - m1YI, mlzl - m2Zt }

ml -m2 m. -m2 ml -ml

Example I. Find the co-ordinates a/the point that divides the join of

(2, -3, /) and (3, 4, -5) in the ratio I ; 3. (Meerut 1985)

Sol; Let the required point be P.

Let P -. (x, Y, z). Then 1 : 3

A. (2,-3,1)

p

B (3,4,-5)

13+32 3+6 9

x=---=--=-

1+3 4 4

1.4+3.-3 4-9 5

Y= =--=--

1+3 4 4

1.-5+3.1

z= =--=--

-5+3

1+3

4

2

U rhcberrec htl lch gesc h utztes M ateri a

10

Golden Solid Geometry

Hence p~{~,

Example 1. Find the co-ordinates of the poin: Ol which the line joining the pocus (4, 1, /) and (I. -1, 6) meets the plane

]x - 2y - z + 1 '" O. (Osmania 1984)

SoL Let the required point be P. Let P divide the line joining the points (4., 3,1) and (1,-2, 6) in the ratio A .:. I. Then

{},,+4 -2A+3 6A+l} p . ...,. A.+"~' A+l

Since P lies on the plane 3x - 2y - z + 3 '" 0

3(>" + 4) _ 2(2)..'''' 3) _ (6A"" I) + 3 = 0

A+l A+l A+l

:::::> 3(A + 4) - 2(2A + 3) - (6A + I) + 3(A + I) "" 0 3A + 12 + 4A - 6 - 6)" - 1 + 3)" + 3 = 0 :::::> 4}" + 8'" 0

)" = -2

{-2+4 4+3-12+1} P...,. -2 + l" -2 + I' -2 + I

P -+ {-2, -7, II}.

Example 3. Find the ratto in which the line joining the points (2, 4, 5).

(3, 5, -I) is divided by the XY.plane. (Agra 80 Type)

SoL Let the XV ·plane divide the join of P(2, 4, 5) and Q(3, 5, -4) in

the ratio k : L Then the co-ordinatcsof !he .pclnt of section are

(3k +2 5k +4 -41 +5.)

"""k+I' k+ I ' ""'"k+T'" ~

Since this point lies on the XY'plane. k ; 1

r, Its z co-ordloateis zero. (2.4.5)

Hence

ie.

(3.5,-4)

Hence

-4k+5 5

---::0 or k = -

k+1 4

Hence the line joining 0, 4, 5) and (3, 5, -4) is divided by the XV· plaae internally in the Mltio 5 : 4.

Example 4. Given P(3, 2, -I), Q(5. 4, -6) and R(9, 8. -10) are

collinear, find the ratio in which Q divides PRo (Agra 1980)

SoL Let Q (5, 4, ~) divide PR in the ratio k : 1

Th(_- CD the co-ordinates of Q) are ~

9k+3 8k+2 -10*-4 R

k + 1 ' k + t' k + 1 is. 2. -4) (9, 8, -10)

U rheberrec htl lch gesc h utztes M ateri a

Co-ordinates

11

But the co-ordinates of Q are given to be (5, 4, -6). .. equating, the x-co-ordinates, we have

or

9k+)

,-- = 5

k+1

4k = 2

or 9k + .3 ... Sic + 5

:. Ie "" 112

Hence Q divides PR in the ratio 112 : I or I:' 2.

Note. Same value of k is obtained when we equate Ihe co-ordinates of y or z, Example 5. A(J, 2, 0). .8(5, J, 2), C(-9, 6, -3) are three poisus forming a triangle. AD, the bisector of angle SAC, meets BC in D. Find the coordinates of the paint D. [Imp.] Sol. Since AD is the internal bisector of L DAC.

DO AD

. . DC ~ AC (Elementary Geometry)

... (1)

A(3,2,O)

AC = J{ -9 _3)2 +{6 _2)2 +(_3_0)2 .J144+ 16+ 9 '" 13.

DO AB 3

.. from (I), DC= AC =13

i.e., 0 divides BC in the ratio 3 : 13.

:. the co-ordinates of the point 0 are

(3(-9)+13(5) 3(6)+13(3) 3(-3)+13(2»)'

3+13 ' 3+13 ' 3+13

Now AB =J{5-3)2+{3-t)2 +(2-0)2 .J4 + 1 +4 =.3

(38 57 17)

,16')6'16 .

Example ·6. Find the ratios in which the join oj the points (J, 2, J), (/, J. 2) is divided by the locus oj the equation

J:r - 72jl + I2Br = J.

Sol. Let P and Q denote the given points (3, 2, 1) and (I, 3, 2).

Let the given locus meet PQ in R, and let R divide PQ in the rali,o Ie. : I.

or

Then, the co-ordlnates of R are

(k+3, 3k+2, 2k+l) k+1 k+lk+1

R

a (1.3,2)

P (3,2,1)

U rheberrec htl lch gesc h utztes Iv! ateri"

:. J,k +3)2 _72(3k +2)2 + 12i.2k +1)2 =.3

\k+1 .k+.l '\k+1

or 3(k + 3f - 72(3k + 2f + 128(2k + If = 3(k + If

or 3(~ + 6k + 9) -72(91<'- + 12k + 4) + 128(4Jcl + 4k + I) =3W+2k+1)

1361<'- + 340k + 136 - 0

2~+5k+2 0

(2k + I) (k + 2) 0

either 2k + I '" 0 which gives k .., -~

k + 2 0 which gives k = -2.

Hence the line PQ is divided by the given surface in the ratio -I ; 2 and -2 : I.

~ Example 7. From the point (I, -I, 2) lines are drawn 10 meet the sphere xl + Y + r = 1 and they are divided in the ratio 2 .' 3. "Prove thai the points of section lie on the surface.

5xl + if' + 5r - 6x + 6y - 12z + 10 ""' O. IV. Imp]

Sol. Let PRQ be any line through P

which meets the sphere

r+y+r=1

12

or

or

or

or

Since this point R lies on 3x2 - 72y + 128z' '" 3

in Q(x" Y" Z,) say

.. X,I + Y,I + %,: = I ... (1)

Let R (e, ~, y) divide PQ in the ratio 2 : 3.

Then

or

or

Golden Solid Geometry

a ~ 2x)+3
2+3
Sa-3
x, 2
~ 2y)-3
S
Y, *+3
2
r = 2z) +6 or %, .. Sy-6
5 2 Putting these values of x" y" %, in (I), we have

(5a2-3r +(5Jl;3r +er;6y '" I

U rhcberrec htl lch gesc h utztes M aleri a

Co-ordinates

13

or 250.2 - 300. + 9 + 25j31 + 301} + 9 + 26yl - 60y + 36 = 4

or 250.2 + 251}l + 25r- 300. + 301} - 60y + 50 = 0

or 50.2 + 5P2 + 5y' - 6a +61} - 12y + 10 = O.

Hence the locus of R (a, p, y) is [Changing o, p, y 10 x, y, z respectively]

5r + Sy + 5zl - 6x + 6y - 12z + 10 >= o which is the required result.

E:la.mpl.e 8. From the point (1. -2. 3), lines are drawn 10 meet the sphere xl + Y + r = 4 and they are divided in the ratio 2 : 3. Prove that the points of section lie on the surface.

Sxl + Sy + Sr - 6x + 12y - ISz + 22 = 0 rlD,lp.1

SoL Please try yourself on tbe same lines as in EumpJe 7 above.

Art. 4. Centroid of tbe triangle

To find the eo-ordinates the eenrrotd of the traingle whose

vertices are (x" Y,• Z,), (~, YI, z,) and

(Xl' Y l'Z,,), I

SoL Let A (x" Y" Z,), B (Xl' Y1, Zl)' C(x,. y,. z,) denote the vertices of the triangle ABC.

If D denotes the mid. point of BC, then its co-ord inates are

A (x" y,. Z,)

( X2 +X) Yl + YJ Z2 + %))' -2-'-2-'-2-

[Art. 3 cor. I) If G (x. y. z) denotes the centroid of the triangles ABC. then G divides

the median AD in the ratio 2 : I. [From Elementary Geometry]

2(X2 ;x3)+L(X1)

x= ' Xl +X2 +x)

2+1 3

B

(11,. Yt. z,l

Similarly

Hence the centroid is

(XI+XZ+X3 YI+YZ+Yl ZI+Z2+Z1.) 3 • 3 • 3 ;.

(Slim o[ X co-ordinates , sum of y co-ordinates

i.e.

3 ' 3

sum of Z c(HJrdinales )

3

IC,T,M.I

IC.T,M,I

U rhcberrec htl lch gesc h utztes M aleri"

14 Golden Solid Geometry

Art. S. Tetrahedron A

Der. A tetrahedron is a figure bounded by four planes not all of which pass. through the same point. Thus ABCD is a tetrahedron.

II has 'C\ = 4 faces, namely the 6s ABC, ACD, ADS and BCD.

It has 'CJ "" 4 vertices namely A, B, C, D each vertex being the point of intersection of three of the four planes.

It has 'C1 = 6 edges, namely the lines AB, AC, AD, BC, BD, CD; each edge arising as the line of intersection of two of the four planes.

/1 has three pairs of opposite edges i.e., the edges which do not meet, namely AS and CD, BC and AD, CA and DB.

The centroid G of the tetrahedron A

ABCD divides the line joining any vertex to the centrold of its opposite face in the ratio 3 : I.. [Frt.lm Stat.lcs]

Thus if 0, is the centroid of ABCD, then G the centroid of tetrahedron ABeD. divides AG, in the ratio 3 : 1.

AG 3

i.e.

--'=-

GOI

c

o

Art. 6. Centroid of tetrahedron [Imp.]

To find the co-ordmates of the centroid of gravity of the tetrahedron whose vertices are (x" y" z,), (x,. Y,. 2::,), (Xl' Yl' zJ> and (x" Y" zJ

Let A (x" Y" Z,)' B(xl, Y" %2)' C(x" )'" z,). D(x" Y._ z.) he the vertices of the tetrahedron ABCD. Then if 0, is the centroid of ABCD, then its co-ordinates are

(~+~+~ h+~+~ ~+~+~)

3 • 3 ' 3

[Art. 4} Then, since G (x, y, z) the centroid of tetrahedron ABCD divide the line A01 in the ratio 3 : I,

Atx" y,. z,l

c

lx" y •• z.>

o (x., Y •• l.)

U rheberrec htl lch ge5c h utztes M ateri a

Co-ordinates

1S

YI + Y2 + Y3 + Y4 %1 + Zl + %3 +:4

Similarly Y = 4 ' z = . 4

Hence the centroid is

(.1+11+1'3+X4 YI+Y2+Y'+Y, ZI+Z;t+Zl+Z.) 4 ' 4 ' 4

i.e.

[sumo/ 0/1 xc:o-ordinates

4 •

sumo/ all y co-ordinates 4

(C.T.M·I

sum =r= ]

[C.T.M·I

Example I. ABCD is a tetrahedron. and A: B: C: D: the centroids

0/ the triangle BCD, CDA., DAB, ABC respectively. Prove thai

(0 AA: BB' CC: DD'divide one another in the rauo 3 : I.

(ii) The centroids of the tetrahedra ABCD and A 'BC'D' coincide. Sol. (i) Let A(xl, YI• ZI)' B(x1• Yl, Zl)' A(x, •. y,. z,)

C(x),.yJ, Zl) and D(xj• Y •• t,) be the vertices of the tetrahedron ABCD, Then the co-ordinates of A the centroid of .6BCD are

(Xl+X3+X4 Y2+Y3+Y4

3 • 3 '

.1:2 +z; +Z4).

[Art. 4)

If G be the point which divides AA' in the ratio 3 : I, then its co-ordinates are

(x .. Y •• zJ

(x •• Y •• l.l

[3(X~+X;+X4 )+[.(XI), 3(~j~)+I.(YI),

3+1 3+1

. 3 (%1 + z; +t'4 )+ '.(ZI)]

3+1

i.e.

(XI +x'2 +xl +X, YI + Yl +Y3 + Y. %1 +:2 +ZJ +Z4) 4 ' 4 • 4'

U rhcberrec htl lch ge5c h "Illes M ateri a

16

Golden Solid Geometry

The symmetry of the result shows that. the points which divide SS', CC', DO', in the ratio 3 : J coincide with G.Thus G lies on each of the lines AA', BB', CC' and DO' and divides each line in the ratio 3 : 1.

Hence AA', BB', CC' and DO' divide each other in the ratio 3 : I (at the point G).

(ii) The centroid of the tetrahedron ABCD is given .by

G(~+~+~+~ ~+h+h+~ ~+~+~+~)

4 ' 4 ' 4

[Art. 6)

... (1) Now A', B', C' and D' being the centroids i\s BCD, CDA, DAB and ABC respectively, we have

A'(X2 +·xl +x4 Y2 + Yl + Y4 z2 +:1 +:4 J.

3 • 3 • 3

and

u(~+~+~ ~+~+h ~+~+~)

3 ' 3 ' 3

Then if G' (ex, p, y) denotes the centroid of tetrahedron A'B'C'o', we

have

X2 +x) +X4 + x) +X4 +xl + X4 +xl+.1"2 +xl +x2 +x3

X '" _----"3'-- "-3 __ . _---=3 __,3o,..__

4

[Viz. sumo! allxc;ordinates {Art. 6)]

X\ +x·2 +X) +X4 4

Similarly p ee YI + Yl + Y3 + Y4 • Y = %! H;Z. H) + %4

4 4

Hence the centroid of tetrahedron A'B'C'o' is

G' (XI +x2 +X) +X4 YI + Y2 + Y) + Y4 %1 +:2 +Z) +Z4)

4 '. 4 ' 4

U rhcberrec htl lch gesc h utztes M ateri a

Co-ordinates

17

From (1) and (2) it follows that 0 and 0' coincide.

Example 2. Show thaI the three fines joining the middle points 0/ opposite edges of tetrahedron meet in a point.

SoL A(xl, 'I' ZI)' so, '1' ZI)' C(X,. Y,. ZI)' D (x.' Y., z) be the vertices of the tetrahedron ABCD.

If L, M be the mid. points of opposite edges AB and CD, th.en we have

L(XI+X2 Y,+Yl ZI+Z2·)

2 ' 2 • 2

A(x,. y,. z,)

and

M(X3+X4 '3 '+ Y4 %3+Z4)

2 • 2 ' 2

Then the mid. point of LM is given by

G[~+~~ ~+~ _Zl_;_Z_~+_Zl_;_.Z_4l

2 • 2 ' 2

~ G(~+~+~+~ ~+~+~+~ ~+~+~+~).

4 • 4 • 4

G, being the mid. point of LM lies on LM,

The symmetry of the co-ordinates of G shows that G is also the mid. point of the lines NP and QR, and thus lies on the lines NP and QR, where N, P, Q, R are the mid. points of BC, AD, BD and AC respectively. Thus G lies on all the lines LM, NP and QR. In other words, the lines joining the mid. points of opposite edges i.e . .LM, NP and QR meet in a point (viz. G).

U rhcberrec htl lch gesc h utztes M ateri a

2

Direction Cosines and Projections

Art. I. Angle between two lines

II is already known 10 the students that the angle between two intersecting or coplanar lines means the acute angle between' them.

We, now define the angle between two non-coplanar or skew lines as below:

Del. The angle between two skew lines is the angle between two lines drawn parallel to them through any point in space.

Thus if PQ. RS be two skew

lines, then the angle which RS makes with PQ is the angle which the line OB ( II in the same sense to RS) makes with the line OA, which is II 10 PQ i.e. L AOB.

L A/S

o A P Q

Note. The angle between a given line and the co-ordinate axes are the angles which the line drawn through the origin parallel to the given line makes with the. axes. [Imp.]

Art. 2. Direction cosines or a line

If n, ~. 1 be the angles thai a given line makes with the positive directions of co-ordinate axes, then cos - a, cos ~, cos '1 are called the direction cosines (or d.c. '5) of the' given line, and are generally denoted by

I, III, n respectively. (K.U. 1986; Kanpur 1986, 83)

Thus 1 "" cos a, m "" cos ~, n '" cos 1,

where a, ~, y are the angles made by the line with co-ordinate axes,

(C.T .. M.I Cor. I. Direction cosines or co-ordinate ales

Since the line x-axis, makes with x-axis, y-axis and r-exls the angles 0, 90~ and 90~ respectively, therefore its direction cosines are cos 0°, cos 90°., cos 90°, i.e. 1,0, O.

Thus the d. c.'s of x-axis are I, 0, 0

Similarly d.c .. 's of y-axis are 0, 1, 0 IC.T.M.I

and d.c. 's of s-axis are 0, 0, I

Cor. 2. If the direction cosine of II line PQ are I, m, n, then the d.c.'s of QP are - I, - m, - n.

18

U rhcberrec htl lch gesc h utztes M ateri"

Let PQ mak.e angles a, 13 • ., with the axes; 50 that

I = cos a, m = cos P. n =- cos ., Through Q draw OA II in the same sense to PQ. so that OA a150 makes angle a, 13. y with the axes.

Now the angles which QP

makes with the axes are the angles .-

which AO produced i.e., OA' makes /~

with the axes.' '1,

But L A'OX = 1800 - a

L A'OY = 1800 - P

L A'OZ = 1800 -.,

.. d.c.'sof QP are cos (1800 - a), cos (180°- 13), cos (1800 -.,) -cos a. -tos 13, -cos j or -I, -m, -no

1m portant Note. In practice, we gtllUrally Ignore the sense of a line and thus I, III. II and -I, -m, -n represen: the d.c. 's af OM and the same line.

Direction Cosines and Projections

and

i.e.,

19

z

y

Art. 3. A useru.! result

If I, m, n be the d.e, '5 of a line UP and OP air, then the eo-ordinates of Pare (Ir, mr, Dr).

Let (x, y, s) be the co-ordinates of z

P and A, the foot of 1. from P on x-axis.

Then by definition p (x. y, z)

x = OA .... (1) [Art. 2J

Now in the It. Ld 60AP

OA

OP = cos a

.. OA= OP cos a ~ r cos a .. From (I),

x '" OA ~ r cos a = rl Similarly y =- mr; z = nr

Thus the co-ordinates of P are {Ir, mr, nr). IC:T.M.l Cor. II follows from above that if (x, y, z) be the co-ordinates of a

point P, such that OP = r, then d.c.'s of OP are

~ !.. !.

. , .

. r r r

y

Art. 4. An important relation between direction cosines

If I, m, n be the direction cosines of a line, tben show that

l' + mJ + n~ =- 1. (K. U. J 984; Kanpur J 983, 81)

Lei AB be the given line whose d.c. 's are I, m, n. Through 0, draw OP parallel 10 AB. so that d.c.'s of OP are also I, m, n. Now cut of OP = I.

U rhcberrec htl lch gesc h utztes M ateri a

Cor. Another form. If a., p, y be angles which a line makes with the

axes. then

I = cos a.. m= cos p, 11 "" cos y Since P + nr + ,r '" I

:. cos' IX + cos! P + cos' y = I This can also be written as

(\ - sinl a) + '(1 - sin! P) + (I - sinl y) .. 3 - (sin2 IX + sin' P + sin' y) =

sinl a. + sin1 P + .sin' y '" 3 - I .. 2 (Agra 84)

Example I. A line makes angles of 4jO and 60° with the positive axes of x and y respectively. What angle does it make with the positive axis of z?

SoL If the line makes an angle y with the positive s-axis, then since the line makes angles 45°,60° andy with the axes; d.c.'s are'cos 45°, cos 60°, cos y.

I I

./2' 2' cos Y

BUI. we know that P+m4,r = I I

-+-+ cos> y '" 2 4

3 cos' Y = 1--=

4 4

. I

C{lS Y =± 2 .: y = 60° or 1200•

Example 2. What are the dt: 's of/he line equally inclined to the axes?

How many such lines are there? [Imp.] (K.U. 1986)

Sol. If a line makes angles a., p, y with the axes, we have here

CI = P ~'y .

:. cos a = cos P = cos y Since P + m1 + ,r = 1

20

or

Then co-ordinate of P are

(I. m; n) [An. 3)

Now by distance formula,

OP=~(I_O)l +(m-O)J +(n-O)1 1= .Jll +m2 +n2

Squaring 1= P + nr + til

P + m1 + Dl - 1 [C.T.M.I

or

Leo, the sum of .squares or the direction cosines or a line is equal to one. I C.T.M. I

or

or

or

or

or

3P = I

Hence the d.c, 's of a line are

Golden Solid Geometry

z

.~B(i.m .n )

A 1

0(0.0.0)

x

y

(proved above) lC.T.M.1

or

I = m = rio f'+ f'+ P=I I 1=±.Jj

U rheberrec htl lch ge5c h utztes Iv! ateri"

Direction Cosines and Projections

21

( I I I )

± ../3' ± .J3' ±.J3

To fmd how m8llY such lines are there, we have to see how many combinations of signs with J, m, n are possible. The various combinations are

+ + + +

+ +

(or + +),) ',' (I, m, n) an,d, (-I, -m.-~)

+ (or + + -) represent the d,c, s

(or + +) of the same line,

(or + + +)

Since there C8Il be only four di,ffennt groups of signs, so there can be fOW' different lines which make equal angles with the axes.

, K

Eumple 3. If a straight line makes an angle of"4 with each of x-

axis and y-axis, then what angle does it make with the s-axis?

(Burclwan 19S3) SoL Let it make an angle y with the z-axis. Then since

cosl a + cosl ~ + cos! Y - I

+

+

+cos~y=

x :. cor 4'

7t

+ cosl 4'

I I -+ - 2 2

=>

K
=> cos y '" 0" cos-
2
K
=> r = -
2 1(

Hence the line makes an angle of "2 with the z-axis,

Art 5. Direction ratios of a line (Def.)

A set of three mlmbus a, b; c, which areproportionai to the direction cosines I, m, n I'UptCliveiy of a line are aIlIed the dircdlon ratios of a lim. We shall write direction ratios as dr. 'so

Caution. Sum of squares of the direction casioes of a line is equal to one whereas the sum of squares of the Clirectlon ratios of a line is not equal to one.

To nadtbc ,actual diredioa coslaes of a llac whose proportional d.e.,', (Leo dfrection nltios) are &!vCD.

U rhcberrec htl lch gesc h utztes M ateri a

22

Gofden Solid Geometty

Let ct b, C be the direction ratios or proportional direction cosines of a lim; and let I, m. n be the actual d.c.'s of the line.

Then

I m n .J11 +nJ +m1 ;="b= ~= .Ja1 +b1 +c I

.Ja1 +b1 +c1 abc

I::, ,m= ,n::. .

.Ja' +61 + cl .Ja' +bl +c' .Ja1 +b1 +c'

'Rule to find the d.c.'s of a Hae wb,ose direction ratios are given Divide

the direction ratios a, b, C each by their sum of squares viz: .Jol +b2 +cl , to get the actual d.c. 's.

H'" Note. d.c, 's of a Unc ~ unique. bul d.r. 's of a line are nOI unique.

If< a, b. c> are d.r.'s of 11 line. thcn< lea. kb. kc > are a.Iso d.r, 's of the same line (if k ¢. 0) .

r·: P+ m1+ n1= I]

B" Note. If a. b. c ~ d.r. ',5 of a line, then a' +b" +c' ~ 1.

Example 4. (0) J, - 2. - 2 are direction ratios of a line. What are Us

direction cosines?

(6) The direction ratios of Q line are 6, 2. 3. Find th« direclion cosines. SoL (a) The direction ratios of a line are I. -'2. -2.

Dividing each by tbcir .SWD of squares viz.

J(l)' + (-2)' + (_2)' ::';1 +4 + 4 :: 3

the actual d.c.·s of the line are 1 -2 -'2

3')'3'

(b) Please try yourself. [ADs. %, ~, %J

Art. 6. Direction ratios of a line join lug two points

(K.U. 1984; Mithi/a 1982) Z

a

~ (x..y •• z,)

p4f····:R

1(,. y" z~

Let P(x •• y •• %.) and Q (Xl' y" %1) be two given points. JoinP, Q and I,et I, m, n be the de.·s of the line PQ. Then if a, ~. y be the angles which the line PQ makes with the axes.

I = cos a, m .. cos ~. n - cos y. Draw PL. QM .Ls on X..axis .. Draw PR 1. QM. so that

L RPQ = Q.

Then PR - LM .. OM- OL'" 1:1- % ••

Now from the rt. Ld &PQR, Y

L

x

U rheberrec htl lch ge5c h utztes M ateri"

Direction Cosines and Projections

23

PR ~ oosa=l= PQ= PQ

Y -Y z -%

Similarly m = IPQ I , n :: IpQ 1

Hence the dc.'s of PQ are

XI -XI ·Yl - YI ZZ -2L

PQ' PQ' ---pQ"

or the d.c.'s are proportional 10 Xz - XI' Y 1 - Y I' ~ - z.l i.e. d.r. 's of the line are x: - XL' YJ - YL' Zz _. ZI.

Remember. The d.c.'s of the line joining two points are

proportional to (i.e. d.r.'s are) -

diff. of X co-ordinates, diff of Y co-ordinate.s, diff of Z co-ordinate.s

rc.T .. M.]

Example 5. Find the de. 's of the line joining the points P (4, 3, - 5) and Q (- 2, l, - 8).

SoL The d.c. 's of PQ are proportionaJ to

-2. - 4, I - 3, -·8 - (- 5) 1< 1:z- XL' Yz- YI• %1- %1>

or -6,-2,-30r6,2,3

.. The actual d.c.'s are (dividin_g the direction. ratios by

~62+22+3z =.)36+4+9 =7

623

:;' :;. :;.

Example 6,' A(-I, 2, -3), 8(5, 0, --6), C(O, 4, -I) are three points.

Show· thal direction cosines of the bisector» of the angle BAC are proportion a/ to 25, 8, 5 and -u. 20, 23.. (Y. I.mp.] Sol. Let the internal and external

bisectors of angle BAC meet BC in the points 0 and E respectively.

Now .

AB == .)(5+ 1)2 + (0- 2)2 +(- 6 +3)2 '" ./36+4+9 ==7

AC =.)(0+ 1)2 + (4 _2)2 +(-1 +3)2

E

'" .)1 +4+4 :z 3.

AD is intema1 bisector of LBAC,

BD AB 7

-=-=-

.• DC AC 3

i.e; D divides BC Internally in the ratio 7 : 3

U rheberrec htl lch ge5c h utztes Iv! ateri a

24

Golden Solid Geometry

· . Co-ordinates of 0 are

[7(0)+3(5), 7(4)+3(0), 7<-1)+3(-6)]

7+3 7+3 7+3

t.e.,

(~ ~ -.5}

2.' 5' 2

AE is external bisector of LBAC.

BE AB 7

_"'-:=,-

sc AC 3

i.e., E divides Be cxtcrnalJy in the. ratio 7 : 3 i.e., internally in the ratio 1 : -3, (Note)

· . Co-ordinates of E are

['(0)-3(5), 7(4)-3(0), 7(-1)-3(-6)]

7-3 7-3 7-3

(-~ 7 .!!)

i.e., 4' , 4

· . d.r.'s of AD are

3 14 - 5

2 -(-1)."5 - 2'2- (-3)

5 4 1

2' s' '2 or25, 8, 5.

d.r.'s of AE are

IS 11

--+1 7-2 -+3

4 ' '4

_.!! 5 23

4' , 4

[Multiplying by 10] 1 See Note. Art. S

or

or

-11, 20, 23.

Art. 7. Lagrange's IdentIty

Ilmp.J

(11+ ml+ DIU11+ m 1+ D·l\ - (II + m m + 011)1

, . r , "'I I , .1 1"l. 1 1-'

- (l,m2 - m,~)l + (m.DI - 0.mJ' + (DII. - I,DI)I. IC.T.·M.I Proof. L.H.S.

= (1,1 + m,1 + n,1) (III + m/ + 11/) - {I.tl. + m.ml + 11,I1J'f

= ',1/11 + I,Jmil + /,J~I + m,l/il + m,'ml1 + m.JI1,_l + 11//11 + 11,lml• + 11/1111- (1,1/11 + m,lm11 + 11,11111 + 2Jl1m,ml + 2m.mll1,nl

+ 2nl'li,i1) .. (I,lmJ! + m.!/.I- 21ll".mJ) + (m,Jn,,1 + mJln/- 2m,mp,112)

+ (11.1'.1 + 1,1~1 - 21lp,n.)

U rheberrcc htl lch gesc h utztes Iv! ateri a

Direction Cosines and Projections

25

= (11m2 - ml/;f + (minI - nlml + (nIl, - Iln;f = R.H.S.

Note. ," "'., ". and '., "'., n, are any tWo sets of numbers and nOI necessarily d.c. '5.

Art. 8, Angle between two lines

IV.lmp.J

~ To find tbe angle between two lines wbose direction cosmes are given (MD.U. 1986. 84; Indore 1983: KU. 1983; Kanpur 1984. 82:

Bhopal 1984; Utkal 1982; Ruhelkhand 1980)

Let PQ. RS be the given lines s

and 11, mi' nr; II' ml, nl, be their Z (~ m, n.) /

direction cosines. LeI e be the angle • B' I ./ a

between them. Through 0, draw OA. R /'

OB parallel to PQ and RS respectively. p

then LAOB '" 9 also. A. (~, m, n,)

Cut off OA = 1 and OB "" 1

:. co-ordinates of A and B are

(I •• mi' n,); (I" mv nl)

x

y

AB'= (11- 11)1 + (m, - ml)l + (nl - "l

= II' + 1/ - 21.tl + mil + m/ - 2mlm1 + nIl + "II - 2nlnl

= (Ill + mIl + n/) + (// + m,> + n/) -2(//. + mimI + n.n1)

= I + I -2(1./, + m.ml + n,n2) ... (1)

1 -: (/1' ml, nl)' (fl' ml, nl) are d.c.·s Now AOAB, by cosine fonnula,

OA I +OB1_ABl

cos e - 20A.OB

_ 1+1-[2-20.11 +mrm1 +nl"z)] 21.1

[Using (m

"" 11/1 + mimI + n,nl

Hence a = Cos-I (ll~+ m,m2+ D.nj. IC.T.M.) ImporUllt Note. If on putting the values of fl' "'I' ", and II' mI' n, we get tht value of cos 9 as negative, then 9 gives the obtuse angle between the two lines. The angle required is generally acute: angle which in this case will be ] 80" - a. But this can also be obtained directly by taking the numerical values of cos e (if il is -ve).

Cor. I. Expression for sln 9 and tan 9. We have.

sinl 9 = I - cos! 9 ~ '1- (Ill + mlm, + n'"l)l

= (Ill + mIl + nll)1 (112 + mIl + ",I) - (Ill + mImI + "Inl

I': 1/+ m/+ "/=1= 1/+ mIl + 'III

urhcbcrre<;htlich gcschiltztcs Maleri"i

26

Golden Solid Geometry

'" (lime m,/Jl + (m,n)- l'I,m,») + (n,ll- l,nlY

I by Lagrange's identity

.. SiD a .. ~L(llm2 -m,ld(Berhampur 1981; KU 198J)IC.T.M.)

siD a ~L(llml -mlll)l

tan a ... cosO"

1112 +mlml + 0101

(Ruhelkhand 1983;' Kanpur 1986) How to write down the result for SiD.

Write down the d. c. 's of the lines in order in two rows I, m, /'I,

'1 1112 n,

Draw diago/'lals mentally as in CToss.muitiplication.

111 /'I I III

I,?f "?f " ?f' m/ ':;jn{ ':;jt/)&1I1:

to get m,~ -/'1,m" lIil-i,fl.z, 1,m, -111,11;' square these and add, then take the square root.

Note. Complete fonnulas for sin e and cos 9 arc given by

and

, J . 2·. ~r.("'1"2 -111"'2)1

Sin 9 - ± L("'I":z -111'":!) • tan 9·~ ± r.l,12

Cor. 2. Condition of perpendiculari.ty

If the lines are .1, then a = 90D

(Bhapal /984)

cos a '" cos 90° .. 0

or I,IJ +m,mJ +D,DJ = 0 [C.T.M.l

Cor. 3. Condition of parallelism. (Bhopal 1984; Berhampur 1981) If the lines arc II, then a .. 0, so that sin a = 0

or ~(l,m, _m,/:)l +(111,112 -/'I1m,)1 +(11,/1 -Il"f = 0

or (I,m, - m,l1)2 + (m,nJ - 11,m,)1 + (n,/: -/,n,,)l = 0

L.H.S, being the sum of three perfect squares, cannot be equal to zero unless each of them vanishes separately.

1,m, -I1I,ll = 0 or I, 111,
.. ~=-
/1 m,
m.nz - n,lIIz = 0 or III, =!i
1111 n"
nll-1,~ = 0 or !!!.=!J..
fl.z /2 ... (iI)

... (iil)

U rheberrec htl lch ge5c h utztes M ateri a

Direction Cosines and Projections

From (I), (il) and (iii),

I, ni, 71, J/,2 +nI,l +n/

7;= ~ = ~= J4.1 +~l +n/

.. I, = 12, m, = m2' D, = Oz. [C.T.M.l Remember. The conditions of perpendicularity and parallelism are deduced from the fonnulae for cos a and sin a respectively.

Art. 9. Find tire angle between two lines whose direction ratios are a" b" c, and a1• hI' cr Deduce the condition for perpendiculUTity and parallelism oftwo lines. (MDU 1984)

SoL Let I" ni" ", be the d.c.'s of the line whose d.r.'s are 01' b, c,'

I =. °1 . nI ~ h, c,

I I I 1 l' I I 1 1 2' ", = I I I 1

'VO, +b, + c, 'VO, + b, + c, 'Va, +b, + C,

Let < fl' m" 11z > be d.c. 's of the line whose d.r. 's are aI' hI' c,.

12=, a1 ,nIl= bz ,711= c1

~all +bzl +Czl ~a/ +b/ +Czl 'Ja/ +b11 +C/

Now Reproduce Art. 8.

:. cos a =1,/1 + m,nlI + 71'''1

Putting the values of ',. ni" "1 and I" nil' 71" we get

t> 81al + blb2 +clCl

cos Q = --;===~~~-?===:l~===

12 2 2/1 2 2

"., +b, +C) Val +b1 +C1

1/ lines UTe 1, a = 900

a,aI + b,b,. + C,CI

•. COS 900= 0 .,

Ja,l +h,' +C,' ~a/ +b/ +c/

.. 8,a1 + b,bl + c,c, ,. 0

if lines are parallel, Reproduce cor. 3, Art. 8

• . I, '" I" ni, ." nil' 71, = 71,

or

.5.. _ Jail +b,1 +C,1 01 JaIl +b,.l +C,1

U rhcberrec htl lch ge5c h utztes M ateri a

28

Go/den Solid Geometry

similarly

!?J. = Ja,~ + b/ + c,~ ~ Ja/ +b,.l +c/

'a,1+,,2+C,1 £L = 'i ...,

C1 Ja22 +~2 +C2l

.!!.=!2=~

a1 bl Cl

and

. Jr.(a,b2 -b,(2)l

sm e = ....:....,='=i'---====-

~ .[i;:

tan e= Jr.(a,bl - alb, )2 1:a,a1

Example 7. Find the angle between the lines' whose direction rauos

are (2, 3, 4) and (I, -2. J). (Bhopal 198.3)

SoL Let the required angle be e, then

cos e = (2XI) + (3X-2) + (4XI)

./22 +.31 +41 Je +(_2)1 +12

Cor.

(C.T.M.]

2-6+4 :It

= ---. =0=C05-

.fi9.J6 2

:It e = - 2

Hence the given lines arc at right angles.

Example 8. If points P, Q arf! (2, 3, -6) and (3, -4. 5). find the angle

thai QP makes with QQ. (Ruhelkhand 1982)

SoL d.r.'.s of OP are 2 - 0, .3 - 0, - 6 - 0

t.e., 2, 3, - 6

d.r.'s of OQ arc 3 - 0, - 4 - 0, 5 - 0

t.e., 3, -4, 5

Let the angle between OP and OQ be e. Then

(2)(3) + (3)(-4) +(-6)(5)

6-12 - 30 -36 18../2 .J49./SO '" 3s.fi = -3'5

U rheberrcc h II ich ges" h utztes ~ I atcrla

Direction Cosines and Projections

29

Hence the acute angle between OP and OQ is coso! (I~~) Example 9. If A, B, C. D are the points (3, 4, 5), (4, 6, 3), (-I, 2,4)

and (/, 0, j). fmd the angle between CD and A..B. (Agra 1980)

SoL d.r.'s of CD are 1 + I, 0 - 2, 5 - 4

i.e., 2, -2,1

d.r.'s of AB are 4 - 3, 6 - 4, 3 - 5

i.e., I, 2, -2

Let the angle between CD and AD be 9. Then

-==(2,f,;X=I)=+=(-,.f2X,.,,2=) +=(=:IX;;-'2=)=

cos 9 = ,

J21 + (_2)2 ... 12 JI2 + 22 + ("':2)1

2-4-2 4

- ---=--

3·3 9

Hence the acute angle between AB and CD is given by

_,(4)

cos -

9

Eumple 10. Prove the direction cosines that the points (a) (1. 2, J). (4, 0., f) and (-2, 4, 2) are collinear.

(b) (1. -2, 3). (2, J, -4), (0., -7,/0) are collinear.

Sol. (a) The given points are

A (I, 2, 3), 8(4, 0, 4), C(-2, 4, 2)

d.r. 's of AB are

4 - I, 0 - 2, 4 - 3

[ Using x2 -x" Y, - Y" %, -%,

or 3, -2, I.

Similarly direction ratios of AC are

- 2 - I., 4 - 2, 2 - 3 or - 3, 2, - 1 or 3, -2, I.

Hence the two lines AD and AC are II [... they have the proponionatc direction ratios] and since they both. pass through the point A, therefore the two Iioes coinctde, Hence A, B, C are collinear .

. (b) Please try younelC.

Example 11. If a variable line in two adjacent positions has direcJions cosines I, m, n; I + 0/, m + am, n + lin; shaw that the small angle 59, between the two positions is given by

(OB)' = (61)' + (om)' + (On)' . [V. Imp.]

(Ruhelkhand 1984, 80; Garhwal 1983; Rajputana 1983; Agra 1980) SoL The d.c.'s of the line in the two positions ire

I, m, n, and 1+01, m+lim, n+lin

U rhcberrec htl lch gesc h utztes M ateri a

30

Golden Solid Geometry

cos 59 '" 1(1 + SI) + m(m + Sm) + n(n + Sn)

1 .: cos 9 = 1112 + mlm2 + nln2 (formula)

12 +m1 +n2 +IBI +m5m+"sn

= I + fBi + m5m + n5n 1 - cos 59 = - (f5l + m5m + n5n)

I',' /2 +m2 +n2 = 1 ... (1)

But 1 - cos 59 = 2Sin2~ = 2( ~r nearly

I',' when B is small, sin. e", B. nellf.IY (From elementary Trigonometry)

which proves the result.

Example 12. A,. B, C, are the points (I. 4, 2), (-2, I, 2). (2, -3, 4).

Find the angles of the triangle ABC.

Sol. The vertices of MBC are A(l, 4, 2), 8(-2, I, 2), C(2, -3, 4), d.r.'s of AB are

-2 - I, 1 - 4, 2 - 2 1 Using XI - xl' Yl - Yl, ZI - Zl

or ...,3, -:J, 0, or I, 1, 0 Similarly d.r, 's of Be are

2 + 2, - 3 - I, 4 - 2

or 4, - 4, 2 or 2,. -'2, 1 1 cancelling 2 and the d.r.'s of CA are I -2, 4 - (-3), 2 - 4

or -1, 7, -2 or 1, -7, 2

2

(59i

'. From (I), _._= - (15I+m5m+n5n) . 2

Now P + ."r. + IT '" 1

and (/+SI)' +(m+5m)1 + (n+Sn)1 = 1

Subtracting (3) from (4),

(0/)1 + (om)! + (&1)1 + 2/S1 +2m5m+ 2nSn = 0

or - (/51 + mOm + nSn) = i[W)l + (om)l +(on)lJ

Substituting in (2),

(5~)1 = 'H(5/)2 + (lim)' + (&1)1]

or (66)2 = (51)2 + (lim)! + (lin)l

... (2) ... (3) ... (4)

A (1, 4 .• 2)

B (-2,1,2)

C (2.-3.4)

U rheberrec htl lch gesc h utztes Iv! ateri a

Direction Cosines and Projections

31

cos A =. (IXl) + (IX-7) + (OX2)

.. .JI +1+0 .JI+49+4

I - 7 -6

= .J2.JS4 = Ji08 .

-6 -I I

.. 6Jj = Jj = Jj (numerically)

-1 ( I )

'"" cos .fj

cos B = (IX2) + 1(-2)+0(1) = 2-2 =0 .JI+I+O.J4+4+1 ..fi:3

B = 90°

cos c= 2(1)+(-2)(-7)+1(2) = 2+14+2 =__!_!_

.J4+4+1./1+49+4 3·.JS4 3·V6

= ~=~

Hence the angles are 90°, cos"! .iJ. cos" (v'f)

B' Example 13. Find the angle between the diagonals of a cube.

[V. Imp.] (Garhwal 1987; Meerut 1986; Jabalpur 1985; Ruhelkhand 1983; Ranchl 1981; Berhampur 1981; Cal.icut 1982; Indore 1982) SoL Take 0, a comer of the cube as origin and OA, OB, OC the three edges through it, as the axes.

Let OA = 08 = OC = Q. Then the co-ordinates of the various points

are

~ 0(0, 0, 0), A(a, 0, 0), )

8(0, a, 0)., qo, 0, a). L(O, a, a). (Note)

M(a, 0, a), N(a, a, 0), P(a, a, a).

The four diagonals are AL, 8M, CN and OP.

U rhcberrec htl lch gesc h "Illes M ateri"

32

Golden Solid Geometry

z

l f---1t--"p

(0, a, a) (a, a,

Ala, 0, 0) X

v

The d.r.ts of the diagonal AL are 0 - a, a - 0, a - 0

. ! Using %2 -XI.' Y2 - y" Z2 -zf

i.e., ~~ a, Q

or -I, 1, I, (cancelling a)

Similarly the d.r. 's of the diagonal 8M are

a - 0, 0 - a, a - 0 or a, -a, a or I, -I, I (cancelling a)

If a be the angle between the diagonals AL and 8M, then

(1)(-1) + (-1)(1) + 1(1)

cos a = .Ji"+'i"+I.J 1 + 1 + I

-1-1+1 -1

J3 .Jj 3 3 (numerically)

-I I .. 9= cos 2"

Similarly the angle between other two diagonals is also the same. u:if'. [Iample 14. A line makes angles a, p, r. B with four diagonals of a cube. Prove thaI

. 4

cw a+ cos' P+ cw r + cw .5 = J' [v. Imp.}

(Bbagalpur 198.3; M.D.U. 1986; GarhwaJ 1986, 81, 83; Gorakhpur 1982; Kanpur 1983, 81; Lucknow 1980; Ranchi 1982; Meerut 1985, 83, 80; Ruhelkhand 1985; Venka 1984; Indore 1983, 84) 501. [Refer to figure of Example 13 above)

U rhcberrec htl lch ge5c h utztes M ateri a

/!)irection Cos~nEls and Projections

33

Take 0, a comer of the cube as origin and the three edges OA, DB, oe as the co-ordinate axes. Then if OA = OB = oe = a. the co-ordinates of the various comers are as shown in the figure.

Then the four diagonals are AL, BM, eN and OP.The d.c.'s of the diagonal OP are proportional to

or

Q - 0, Q - 0, Q ,- 0 Q, Q, Q or 1, I, 1

I I I actual d.c. 's of OP are .JJ '../3' J3

-I I 1 Similarly the d.c.'s of diagonal, AL are -Ii' -Ii' .fj:

I -I I the d.c.'s of diagonal BM are .,[3' .fj' .fj :

, ) 1 I

the d.c.'s of diagonal eN arc .,[3' .J3' - .fj'

and

Let, I, m, n be the d.c.'s of the given line which makes angles a,p, 'f, 5 with the four diagonals OP, AL, BM, and CN respectively (say).

1/,1 I 1('1 ')

cosa=-' +-·m+-·n=- +m+n

.f3 .f3 .,[3 J3

I

Similarly cos p = Jj (-i+m-t;,n)

I

cos y = Jj (I-m+n)

I

cos S = r::: (/+m- n)

· ... 3 '

and

cosJ a + cos' P + cos' 'f + cos'S

I 2 1 1 I 2

= _;_(/+m+n) +-(-l+m+n) +-(/-m+n)

3 ' 3 3

I .,' r +mi +n' = I

U rheberrec htl lch ge5c h utztes Iv! ateri a

34

Golden Solid Geometry

~ Example IS. TIw edges of a rectangular porallelopiped ore a, b, c, show that the angles between the four diagonals are given by

co.r! -

al ±b1 ±c1 a1 +b1 +c1

[V. Imp.]

(Kanpur 1987 82; Meerut 1984; Ruhelkhand 1982) z

y

Sol. Take 0, a corner of the rectangular paralleloplped as the origin

and three edges OA, DB., DC through it as the IIX.CS.

Then the co-ordinates of the various corners are as shown is the figure.

The four diagonals are AL, BM, CN and OP (Note)

The d,c, 's of AL are proportional to

0- a, b - 0, c - 0 I Using X2-X1, Yl-YI' Zl -ZI

- a, b, c.

Similarly d.e .. 's of BM are proportional to a. - b. c d.c.'s of eN are proportional to D. b. - c

and d.c.'s of OP are proportional to D, b, c

If a is the angle between the diagonals OP and AL, then

o(-a)+b(b)+c(c) _01+b1+C1

rosa'" = .

.Jal +b1 +c2 ,J02 +b2 +c2 02 +b1 +c2

... (1)

(al _b2 +c2)

Similarly angle between OP and 8M'" cos" 2 b1 '

a + +C'

... (2)

U rheberrcc htl lch gesc h utztes Iv! ateri a

l3irection Cosines and Projections

35

Similarly angle between OP and CN = cos"

.,.(3)

Similarly angle between At and 8M" cos"

[when acute angle is taken}

This angle is the same as the angle between OP and CN. Similarly we can show that all other angles between other two diagonals are repeated and we get only three different angles as given by (I), (2) and (3). Hence the angles between four diagonals are given by

_ (a1tbl ±Cl)~ cos!

a2 +b2 +cl

Note. The signs in the numerators cannot be all +ve, .if they arc all +Ve,

then me angle

_1.(a2+b2+c2) -110

= cos . a2 +b2 +c2 = cos . =

i.e. me two diagonals will be II. which is impossible as it is clear from the figure.

Exa.mple 16. Find the angle between the two lines whose direction

cosines are given by the equations.

(a) I + WI + n = 0 and F + "11_ rr = 0 (Raj. 1983; Calicut 1984)

(b) 21 - "I + 2n = 0 and "In + nl + 1m = 0

(c) I + WI + n = 0 and 21 + 2m - nm '" 0

(d) I + 3m + 5n = 0 and 2mn - 6n/ - 51m = 0 (e) I + m + n = O. 2mn + 3/n - 51m = 0

(Meerut 1983; Garhwal 1985)

if) 1+ m + n = 0, 21m + 2nl- mn '" 0 (Meerut 1983, 85)

SoL (a) d.c .. 's of the two lines are given by

I + m + n .. 0 (.1)

and P + m"! - n2 = 0 (2)

From (I), I = - (m + n)

Putting this value of I in (2), we get (m + n)2 + m2 - n2 = 0

or 2m2 + 2nm '" 0

or m(m + n) '" 0

U rhcberrec htl lch gC5C h utztes M ateri a

36

i.e.,

.. Either m ~ 0 or

0.1 + m + O.n '" 0 i.e.,

Also from {I)l + m + n = 0 Also

Solving by cross-multiplication,

I m n

--=--:;-

Go/den Solid Geomefty

m+n~O 0.1 + m + n = 0 /+m+n""O

Solving by cross-multiplication,

I m n

-=-=-

1-0 0-0 0-1 I-I 1-0 -I
I m n I m n
or -=-=- or -=_=_4
0 -I 0 -1 Hence the d.c.'s of the two lines are proportional to (i.e. d.r. 's are) I, 0, -I and 0, I, -I

If 9 be, the angie between the two lines, then

leO) +0(1) + HX-I) 1

cos 9 '" ./1 +0+ I ,/0 + I + I '" 2"

(b) The d.c.'s of the two lines are given by 2{ - m + 2n = 0

and mn+nl+lm=O

From (I), m = 2(I+n).

Putting Ibis value in (2), we have 2n(n + I) + nl + 2/(/ + n) = 0

or 2n2+ 2n1 + nl + 2P+ 2In = 0

or or

2P+ Snl + 2,r= 0 (2/ + nXI + 2n) = 0 2/+n=0 or

either

i.e:

2l + O.m + n ee 0 Also from (I), 21 - m + 2n =0 Solving by cross-multiplication,

m n

--=--=--

0+1 2-4 ·-2-0

I m n

or

--:=-::;;-

-2 -2

I .m n

or

-I 2 2

... (1) ... (2)

1+2n=O

i.e. I + O.m + 2n = 0

Also from (1), 2/ - m'+ 2n ;" 0

Solving by cross-multiplication,

I m n

--=--:c:--

0+2 4-2 -1-0

I m n

-=~=--...,....

or

2 2 -I

Hence the d.c.'s of the two lines are proportional to i.e. d.T.'s are - I, 2, 2 and 2, 2, - 1

If 9 is the angle between two lines, then

_ (-IX2)+2(2)+2{,-I) -2+4-2 =0

cos 9- ./1+4+4./4+4+1 9

U rhcberrec htl lch gesc h utztes M ateri a

Direction Cosines and Projections

37

:. e = 900 i.e; the two lines are perpendicular. (c) The d.c. 's of the two lines are given by /+m+n=O 2/+2m-mn~0

From (I), f = "'" (m + n)

Pulting this value of I in (2), we get -2m-2n+2m-mn=0

or -2n-mn=0

or - n(2 + m) = 0

.. Either n= 0

i.e., 0.1 + O.m + I.n = 0

Also from (1) I + m + n = 0 Solving by cross-multlpflcatton,

... (1) ... (2)

or

m+2=0

or

I m n

.. m = -2.

:. (1) becomes I - 2 + n = 0 It is satisfied by I = I, n <= 1

C·,. I - 2 + I = 0)

m n

--=--"'--

0+1 1-0 0-0

Putting I = 1, m co -2

lind

n = t in (2),

-1 0

weget2-4+2=OorO=0

:. our choice ofl = I, n = I is correct.

or

Hence the d.r, 's of two lines are

-I, I, 0 and I, -2, I.

If e is the angle between the two lines, then

(- 1)(1) + 1(- 2) +0(1) . - 3

cos a '" . JI+1+0.Jl+4+1 = .J2..f6

- 3 - 3-13

cos e = J1i = 2../3 = -2-

e.J3 .

:. cos = T{Nwoencatty)

(d) Please try yourself. fe) Please try yourself.

(/') P lease try yo'OrseJr.

Example 17. Find the de. 's I, m, ~ of two lines which are connected by

the relations / - 5m + 3n .. 0, and 7P + 5nr - 3IT .. 0 (Bhopal 1984)

U rhcberrec htl lch gesc h utztes M ateri a

38

Golden Solid Geometl}'

SoL I - Sm + 311 '" 0

7P+ Sm~- 3~ == 0

From (1), 1== Sm - 311

Put this value of J in (2), we get

7(Sm - 3n)2 + Sm1- 3~ = 0

::::) 18.1111- 210mn + 60~ '" 0

::::) 6",.- tmn + 2~ == 0

::::) (3m - 2nX2m - n) '" 0

::::) Either 3m - 2n == 0 or 2m "'" n == 0

when 3m - 211 '" 0

... (1) I Given ... (2)

=>

m n

_ == - "'). (say)

2 3

m==2l.., n= 3).

:. From (I) 1- 10), + 9), == 0::::) / == ).

I m n

I =>-==).

I

-=-=-

I 2 3

Hence the direction ratios of one line are I, 2, 3

. . d.c .. 's of one line are

I 2

3

::::)

Jtl+21+3Z' J12+2z+3z' Jlz+21+3z

I 2 3

Jl4'Jl4'Jl4

2m-n==O m n

._ .. - .. '" (sav)

I 2 v

m==).

n == 2}'

From (I), I - SA. + 6A. == 0 1== -}.

I

- - A.

-I

t.e;

when

::::)

m II

= -=

-1

2

Hence the d.r.'s of the second I.ine are -I, I, 2

•• Th.e d.c.'s of the second line are

I I 2

- .[6' ../6' 76'

U rheberrec htl lch gesc h utztes M ateri"

Direction Cosines and Projections

39

IGJ" Example 18. Show that the straight lines whose direction cosines are given by the equations

"I + vm + wn .. 0, all + bm1 + CIT ~ 0 are

(i) perpendicular if u1(b + c) + v!(c + a) + wZ(a + b) = 0

(Meerut 1986, 84; K.U. 1982; Osmania 1988)

,,2 v2 w2

(i,) parallel if -+-+-=0. abc

(M.D.U. 1983; K.U. 1982; Kanpur 1988; Osrnania 1988) (V. Imp.'

SoL The d.c.'s of the two lines are given by u[+vm+wn=O

and of + bnr + en: .. 0

... (1) ... (2)

_ (UI+'wvm) Eliminating n from (I) and (2) by putting n =

From (I) in (2), we have

all +bm2 +c( UI:vmJ "= 0

or ~12 + bw2m2 + c(~/2 + ,,2m2 + 211\'1m) = 0

or ['(awl +cu1)+2uvc/m+m1(bwl +cv1) '" 0

Dividing throughout by m1, we get

12 I

-, (awl +CU)2 +2uvc-+(bw' +cv2) ",0

m m

which is quadratic in Vm.

If '1' mI' nl and 11' mI' n1 are the d.c.',s of two lin,es, 11 I~

then ~' ~ are the roots of (3).

... (3)

or

av1 + bu' (By symmetry changing a, b, c and II, v, w in cyclic order) ... (4)

= k (say)

(I) Now the two lines will be .1 if

U rheberrec htl lch gesc h utztes Iv! ateri"

40

Golden Solid Geometry

or or or

if k(bw1 + ev') + k(aw' + CUI) +k(av1 + bu') = 0 bw' +CV· +mv' +CU' +m,l +bu' '" 0

if (b + c)ul + (e + a) v' + (a + b)w1 = 0 (i1) The two lines will be parallel if

.!r..=!!!.L=!!.L

»I, m, n,

I From (4) I cancelling k

or

J__ = .!L, or if the quadratic (3) has equal roots.

»I, m,

if the discriminant of (3) is equal to zero.

if 4c'u'v' -4(aw' + cu'Xbw1 +cv') = 0

b' - 4ac = 0

if

or or

or

if if

-w'(abw' + bcu' +cav1) = 0 abw' +bcu' +acv1 = 0

or

or

01 v1 w1

if -+-+- = 0

abe

[on dividing throughout by abc]

How to write down the result (4) by symmetry? Change I, »I, 'I; a, b, c and u, v, w to get the next number by symmetry. [V. Imp.]

~ Example )9. Prove that the straight lines whose direction cosines are given by the equations

al + bm + en = 0 and fmn + gnl + him = 0

are (I) perpendicular if f +K+!!. = O.

abc

(KU. 1986, 83; Meerut 1985, 84, 82; Allahabad 1982; Ruhelkhand 1981; Indore 1983)

(iI) parallel if ..{ri +.fbi +.Jd, = O.

Sol. The d.c. 's of the lines are given by al+bm+cn=O

and fmn + gnl + him = 0

(V. Imp.] [Garhwal 1984]

... (1) ... (2)

From (I)

Putting this value of n in (2), we have

-fm (a/:bm)_gl( a/:bm) + him = 0

or

U rheberrec htl lch gesc h utztes Iv! ateri a

Direction Cosines and Projections

41

or fm (al + bm) + g/(al + bm) - helm = 0

or agP + Im(af + bg - eh) + fbm% = 0

Dividing throughout by m%, we have

12 I

ag-1 +-(af+bg-eh)+bf=O ... (3)

m m

Then if I" m,. n, and '%' ml, n1 be the proportional d.c, '5 of the two

lines, then .l., !:L are the roots of (3).

~ ~

1'/2 hi

-- "" product of the roots = - m,~ ug

= 110

g I b Note this step

I To bring symmetry between

or

f-+a,g-+h [by symmetry]

change f, g, h and a, b, e in cyclic order

1,/, m,~ n,n,.

fla = glb = hie

= k (say)

(I) Then the two lines will be .1 if 1'/2 + m,nI, + n,Ja, = 0

or if k(fJa + gIb + hie) = 0 i.e. if I +.!b +~:: 0

a e

(il) Again the two lines with d.c.ts I" m" n, and 11' mI, nt' will be II, if

or

if

i=!!i=!!L

'1 m: n:

.l.=!:L

m, m1

[From first two members]

or or or or

if the roots of (3) are equal

if the discriminant of (3) is zero

if (af+ bg - ch)'- - 4agbf= 0

if a1f1 + lJlgl + c'lr - 2abfg - 2bcgh - 2cahf = 0 But the given result to be proved is

ra.r + Jbi +.Rh ;0 0

.Jai + Jbi = --Jd,

af +bg+2~abfg '" ch

... (4)

or

Squaring

Urheberrechtlich qeschutztes Materia

42 Golden Solid Geometry

or af + bg - en = -2JQii

Squaring again, (af + bg - chY = 4abfg

or 01 f2 +b1g1 +c~hl-2abfg-2bcgh-2CQhf = 0

which is the same as in (4), proved above.

Hence the result

Example 20. Prove that the two lines whose direction cosines are connected by two relations.

al +bm+cn = O. u]2 + vm2 + wn2 = 0 are (I) perpendicular if

01(v+w)+b1(w+ u)+c1(u+v) = o,(Meerut ]984, 86; Osmania ]988) and (il) parallel if

02 b2 c1 -+-+-=0.

u v w

Sol. Please Cry younel'.

(Hi.nt. Refer to ~ple 18. where a. b, c and It, Y, W are interchanged.] E.umple 21. Find the direction consines of the line which is

perpendicular to the lines with direction cosines proportional to I, -2, -2 and 0, 2, I.

Sol. lei I, m, n be the d.c.'s of the line .L to the given lines

Thus

1(1) + m(-2) + 11(-2) '" 0 or I - 2m - 2n = 0

and 1(0) ... m(2) + 11(1) '" 0 or 0.1 ... 2m + n - 0

Solving by eross-multiplicatlon, we have

I m n

--=--

or

-2+4 0-1 2+0

1m"

2' = :(='2'

.. The d.c.'.s are proportional to 2, -I, 2.

Hence the actual d.c.'s are

.2 -I 2

3'3'3'

Eumple 22, lL• mJ' "I; 11, m~, n2 are the direction cosines of twa mutually perpendicular lines. Show that direclio.n cosines of th« line perpendicular to both of them are

"'Jn1-n,mZ' n,11 -JI~' I,m, -m,[l' [Imp.] (M.D.U. 1987)

SoL If I, m, n be the d.c. 's of the line 1. two given lines. then

IlL + mmL + ML = 0

"l ... mml + nn1 = 0

U rhcberrec htl lch gesc h utztes M ateri"

Gjrection Cosines and Projections

43

m

n

---=----

nll - lin,. I,~ - mill

(from Elementary Algebra)

__ 1_=_1_=1 - sin e sin 90°

where e = 90° is the angle between th.e given lines.

.. I = m'"l- n,m,. m '" n/2. - lin,. n ." 1,m2 - m/,

i.e., the d.c.'s are

min. - n,m,. ",II - ',n,. I,m. - m,ll,

Example 23. If I" m" n,; t, mr n1 and I" m" n) are the direction cosines of three mutually perpendicular lines, then show that the line whose direction cosines are

11+12 +/3 ml +m2 +m3 ". +112 +n3

J 3 and 3

makes equal angles with them. (Ravishankar 1984; Avadh 1982)

SoL Since I" m" n,: I,. m", n, and I" m" "1 are the direction cosines

of three mutually perpendicular lines, we have

I,l +m,2 +11,. 1=1 )

III +~1 +n11 = I

l,l +m,1 +n/ = 1

1,/) +m,m" +n,~ = 0 ) Ill, +m"mJ +1I:zn, == 0 1,1, + m,m, + nJn, '" 0

Let the angle between the lines whose direction cosines are I" mi' n, and

be O. Then,

cos 0= Ile,+/i+/l)+m,(ml+~l+ml)+n,(nl+i+nJ)

U rhcberrec htl lch gesc h utztes M ate,; a

44

Golden Solid Geometry

(/,2 +m,2 +1'1,2)+(/,/2 +m,in:! +n,nz) +(/,/] +nr,~ +n,n)) 3

1+0+0

---':=:._

3 3

=> e = cos" GJ

each.

1 Similarly. we can prove that the other angles are also equal to coso, (")

Example 24. Show that three concurrent lines with direction cosines

t, nI" I'll; 12' ml' 1'12 and I)' ml, 1'1) are coplanar if

II nil I'll

12 m2 1'12 13 nlJ nJ

(Allahabad 1986; Agra 1984; Meerut 1981) Sol. Lei OA, DB, DC be the given three concurrent lines with d.c.ts

I" m" 1'1,: 12' ml' I'll and I), m" If" respectively. N ..

Now if these lines are coplanar, then these lines have a common normal.

Let ON be the common normal and

let its d.c. 's be I, m, n.

Since ON .1 OA, OB, DC.

. . II, + mm, + Ifn, '" 0

III + mmz + 1'11'1, = 0

II] + mml + 1'11'11= 0 Elimination I, m, .1'1, determinatically

from the above equations,

we have

/1 mIn, I

I, mI. n, =0 I] ~ I'l,

'" O.

(Imp.]

a

B

A

Writing co-efficient of I in one column, and co-efficient of m and n in second and third columns

which is the required condition.

Example 25, I,. m" I'l, and 12' ml' I'll are the direction cosines of two concurrem lines. Show that the direction cosines of the lines bisecting the angle between them are proportional to

II ± I" m, ± m" 1'1, ±", . (Garhwal 82)

SoL Let the two lines ~A. DB be concurrent at O. Take points .P, Q on these two lines such that OP = OQ = I. Also take a point P' on AO produced such that OP'''' OP = l. Join PQ and P'Q.

U rhcberrec htl lch gesc h utztes M ateri a

Direction Cosines and Projections 45

p A (I"m"n,)

Let M. M' be the mid point of PQ and P'Q. Then OM and OM' are th.e required bisectors.

Now the co-ordinates of the points P, Q and P' are P(I" m" n,), Q(/,. nit' nl)' P'(-l" -ml' - n,)

I By formula (/r, mr, nr) .. Co-ordinates of M, M', the mid. point of PQ and .P'Q, are given by

M(/l+ll ml+m2 n1+n2)

2' 2 ' 2

and M' (" ; ". mz; m,. n2; H, )

Hence the d.c.'s of the bisector OM are proportional [0

II +/) -0 m, +m) -0 n, +~-O

2 • 2 • 2

I, + l~ m, + ml' n, + H,

Similarly d.c.'s of OM' are proportional 10 i1-i" m,- m" n1- n,

or I, - I~ m, - m,. n, - n1 .•. (2)

Combining (I) and (2), the d.c.ts of the bisectors are proportional to I, ±i1• nI, ±mz, H, ±1Iz.

Example 26. 1[ I"~ m" n, and I •• m" n. be two directions inclined at an angle a, show that actual direction cosines oj the direction bisecting them

or

... (1 )

are

!..(11 +12)sec Of 2. !..(ml +m2)sec (}f 1, !..(nl +nl)sec Ol2. [Imp.] 222

(M.D.U. 1987, 83) SoL Through the origin 0, draw two 51. lines AOB and COD parallel 10 the given intersecting lines so that LBOC = 9.,

Cut off Of = OQ = I.

d.c.ts of OB are t, m" n,

U rhcberrec htl lch gesc h utztes M aleri a

46

Goldan Solid Geometry

Co-ordinates of P are

o

a

I Using (Ir, mr; nr)

p

Similarly.co-ordinates of Q are (lZ' mZ' nl)'

If OM be the bisector of LeOB. then M is the mid. point of PQ and hence its co-ordinates are

,

,

'M

---r---

o

A

Now d.c.'s of OM are proportional to i.e; d.r.'s are

[(II +12) (m, +111:0) (II, +":2)J

2' 2 ' 2

a

c

I Using X;; -Xl' Y: - y,. %: -Z,

or I, + 1Z' m, + mZ' n, + nl ... (1)

Since the two lines OB and DC are inclined at an angle e,

cos e = 1,11 + m,m: + n.n:

Also '" ~(Jl +IJl +(m, +mS +(11, +n1)1

... (2)

I··: 112+m/+ .. n12. =1=/22+"'22+n/ and using (2)

= J2{1 +cos e) = ~2(2,cos2 e 12) = 2 cos S 12.

From (I) and (3), the actual d.c.'s of the bisector OM are

. .. (3)

" + 'I ml + Ill:! II, + III

e' S' e

2cos - 2eos - 2cos- 222

I a I e I e

or "2(11 + /2) sec "2' "2(ml +"'2) sec "2' "2(111 +112) see "2'

E~mple 27. 0, A, B, and C are four points not necessarily lying In the same plane and such that OA J. Be and DB J. CA. Prove that OC J. AB. (Kanpur 1988)

U rhcberrec htl lch ge5c h "Iltes.. M ateri a

Direction Cosines and Projections

Sol. 0 ~ (0, 0, 0)

Let A ~ (x" '" z,)

B ~ (xl' Y:!" Zl)

C ~ (x" Y" z,) Direction ratios of OA are x,- 0, Y,- O. t,- 0

i.e., x" Y,. z,

and Direction ratios of BC are x, - x". Yj - Yl' Z, - Zl OA.l BC

.. x,(x, - x,) + Y,(Y, ;- Y1) +z,(z, - Zl) = 0

Again, Direction ratios of OB are x2- 0, Yc 0, %:- 0

i. e . ., x2' y:z~ Zl

,and Direction ratios of CA are x, - x" Y, -.Y,. l, - z, DB .1 CA

. , xix, - x,) + y/y, - y,) + ll(1., - z,) '" 0

Adding (I) and (2), we get

X'I(X, - Xl) + Y,(Y, - Y1) + zil, - ZI) = 0 ~ x,(x, - x,) ... yly1- Y,) + Zll,- %,) = 0 Now Direction ratios of DC are

x, - 0, Y, - 0, Z, - 0

i.e; x,. Y;. z,

and Direction ratios of AB are

47

... (1)

... (2)

... (3)

X,- x,, Y:- Y" ZI- z,

If DC .1 AB. then

x,(x, - x,) + Y,(Y, - Y,) + :Z'(%1- :,> = 0 which is true by (3)

Hence DC .1 AB.

Example 28. Prove that if two pairs oj opposite edges of a tetrahedron are perpendicular, then third pair is a/so perpendicular. (Ruhelkhand 1984) (Imp.) SoL Let OADe be the tetrahedron with OA .1 BC and DB .L CA. To prove that CO .1 AB. Take one vertex 0 as the origin and any three mutually

.1 line thro' it (not shown in the figure) as the axes. A (x" y,. z,)

Let A, B, C be (x" y" t,), (xl' Y:!' Zl)' (xl' Y" Zl) Then direction ratios of OA are x, - 0, Y, - 0, z, - 0 I Using x2 - x" Y, - Y,• %,- z,

or x" Y" %" Similarly d.r, 's of BC are

x, -, Xl' Y, - YI, ,Z, - l, B

OA .1 BC (given), tx .. y" z,)

.. ''',(xl - x,) + y,(v,- Y,) + %,(z, - %,) = 0

c (x,. Y.,~

... (1)

U rhcberrec htl lch gesc h utztes M ateri"

48

Golden Solid Geometry

Again d.r, 's of OB are Xl' Yl, z,

and d.r. 's of CA are x, - Xl' Y,- Y3, Z, - z)

As OB .1. CA (given)

.. xix,- .l) + Y'(Y, - y,) + ziz,- tJ) = 0

Adding (1) and (2), we have

xlx, - Xl) + Y,(y, - Y,) + Z)(Z, - z,) = 0 d.r.ts of OC are xl' Yy Zl

and d.r.'s of AS arc x, - x" Y~ - Y" z, -z,

or x,- xl' Y,-Y,. z,- Zl

Now OC.l. AS if

xl(x, - x,) + y)(Y, - Y,) + z)(z, - Z'l) = 0 which is true because of (3).

Hence OC .L AB .• which proves the result.

Caution. The student is advised not to take OA, OB, OC as axes (because they are not mutually perpendicular).

Ex.ample 29. If in a tetrahedron DABC,

OA' + BOl "' 081 + OA1 = ()(Y + AB'

... (2)

... (3)

then its pairs of opposite edges are at right angles.

Sol. Let 0 (0, 0, 0). A (x" YI, z,), B (xt Yr z~). C (xl' Y)' z) be the vertices of the tetrahedron OABC

Since

OA'+ BC'= OB'+ CA'

(X12 + YI2 +ZI2) +(X2 -xJ)2 +(Yl - YJ)2 +(Z2 -Z3)2

'" (x/ + Y/ +z/) +(X, -XJ)l +(Y1 - Yl)2 +(Z1 _Z)2 Z(X12 + y,2 +Z,2)-Lx2X3 = L(x/ + y,2 +z/)-2LX1XJ

2Lx,xJ -2 Lx2xl = 0 2LXJ(X1-Xl) = 0

(given)

or

or

. or

i.e., x/-t, - Xl) + YI(Y, - Y1) + z,(Zt - Z'l) = 0

Now d.r.ts of OC are Xl - 0, Y, - 0, z, - 0

and d.r, 's of AB are x1- x" Y1- Y" 21 - Z,

or x, - X" Y, - Y,. z - t,

Now OC .L AB Jf x,(x,- x,) + yly, - y,) +zlz, - z,) = 0 which is true by (1). Hence OC .l. AS.

Similarly taking the other conditions, we can prove that OA .l. BC and OB .1.. AC.

.• .(1 )

Example 30. Two edges AB, CD of a tetrahedron A8CD are at right angles. Prove that the distance between the middle points oj AC and 8D is equal to the distance between the middle points of AD and Be.

(Kanpur 1985)

U rheberrcc htl lch gesc h utztes Iv! aleri a

Direction Cosines and Projections

49

Sol. Take A as the origin and let the other vertices be B(x,• YI' %1)' C(x:, Y:, %,), D(xl• Yl, %1)

d.r.'s of AB are x1- 0, YI- 0, ;1- 0, i.e., XI' Y" ZI and d.r.ts of CD are x2 - Xl'

Yl- Yl' %,- Zl

Since AB j_ CD (given)

•. XI (Xl-Xl) + Y1 (Yl - Y1) + ZI (Z2 - Zl) == 0 ... (1)

If 1" M, .P, Q be the mid. points of the edges AC, BD. AD and BC

respectively., then we have

and

or if

or

or

or

or

c (x,. 'I., z.e!

M (XI+XJ YI+YJ %1+Zl)

2 ' 2 ' 2

p (.~ 13 "3) 2' 2' 2

Q (XI; x!, YI; Y2, %1; "2 )

We are to prove that. LM = PQ. This is so if LMl '" PQ2

(XI H; -x! J +(YI + ~ - Y2 Y +( %, +~ -Z2 J

if

i [(XI +Xl-x2f -(XI +x1-xS] + + == °

1: [(XI +x,-xS -(XI +X2 _Xl)2] == 0

1:(4%I%j -4x,x2) = 0 or if 1:xl(x, -xJ = 0

LX,(x1-x) = 0 I On taking-ve sign

if

if

if

or if XI (X2- Xl) + 'I (Yl - Y1) + ZI (Z2 - Zj) = 0

which is true by (I).

Hence LM = PQ,which proves the result.

Example 31. Prove thai the area 0/ a triangle whose vertices are (0, 0, U); (x" Y,. z,) and (x:, Y:, %2) is

U rheberrec htl lch ge5c h "Illes Iv! ateri a

50

Golden Solid Geometry

B(""Y •• zJ

e

a M

(0.0,0)

Sol. Area of triangle OAB

OA·BM OA·OBsin e

2 2

..!. Jr.x 2 Jr.:c 2 JL(Y,Z, - YlZ,)l 2 ' ~ ~~ vl..X, Vl..X2

A (x" Y .. z.,)

Arl. 10. Projection on a line

(0) The projection of & point on a line is the foot of .i from the point on the line.

,P ,

,

,

,

,

,

,

,

A P' B A p' C' B

Thus projection of P on AB is P'.

(b) The projection of a segment or a line is the line joining the feel of perpendiculars from tts ends on the fine. Thus P'Q' is the projection of . PQ on the line AB.

(c) Length or projection

P'Q' '" PI? . PQ cos a .{C.T.M]

where e is the angle wnrch the line PQ makes with PR i .. e. with AB.

U rheberrec htl lch gesc h utztes Iv! ateri a

Direction Cosines and Projections

51

Note llial if PO .1 AB, the projection of PQ on AS = 0, as 9 ;: 90°. [Imp.)

Art. 11. To prove that tbe projection of the join of two points (x" Y" Z,), (X" Yz• 21) on a line whose direction cosines are I, m, n is

I(x,- x,> + m(y,- Y,} + n(Zz- z,). LetP (x" Y,• Z,) and Q (x" Y" I,)

be the given points and AB the line whose d.c.'s are I, m, PI. Draw PP', QQ' .Ls on AB.

Then P'Q' is the projection of PQ on AB,so that

P'Q'= PQ cos £) ..• (1) A

where a is the angle which PQ makes

with AB.

a ~(x.,y .. zJ

~ ~

(~" Y .. l,): ~

· .

· .

· .

· .

· .

· .

· .

· .

P'

Q'

B

Now d.T.'S of PQ are x,-x" Y,-Y" 2:,-Z"

Dividing each by J<x, -x,)' +(Y, - y,): +(l:! -z,)' = PQ

X, -x, Y, - Y, Z, -Z, the actual d,c .. 's of PQ are PO' PQ' .--po-

Also the d.c.'s of AB are I, m, n

cos £) = I( Xlp~XI)+ m (Y:p~Yl) + PI (Zlp~ZI)

(given)

[Using cos a = I,~ + m,"':l + PI,n,j

"" ~Q [/(X, -x,)+m{Yl-y,)+n(z, -Z,)] ... (2),

Putting this value of cos £) in (I). we have

P'Q' = I(x, - x,> + m(Yl - Y,) + n(z,. - z,) le.T.M) Note. The above result is very important and is very oftenly used in Ihe next

work.

Example 32. The projections oj a line on the axes are 3, 4, /2. Find the length and d.c. 's oj the line.

(Agra 1981 Type; Bihar 1981 Type; Berhampur 1981 Type) Sol. Let the ends of the line

be P (x" Y" z) and Q (Xl' J'" %1)'

3 -= projection of the line PQ on x-axis

= I(x,- x,) + m(v,- Yl) + n(zl- Z,) = [(X2 - x,) + 0(", - Y,) ... 0(%,- %,)

[ .,' d.c.'s of r-axis are I, 0, 0]

or

= x,- x,

-= x, - x, -= 3

U rheberrec htl lch gesc h utztes Iv! ateri a

52

Golden Solid Geometry

4 = projection of line PQ on y-axis

= O(x,- xl).+ l(y, - YI) + 0(:, - :1)

[.: d.c.'s of j-axis 0, I, 01

= Y'-YI

or = Y1-YI = 4

also 12 = projection of line PQ on z-axls

= O(x, - XI) + O(y,- YI) + 1(:, - :1)

[.: d.c.'s of r-axis are 0, 0, I]

= :,-:1

or ~- II = 12

Length PQ =; J'(x-2---x-1 )-:-1-+-(Y-'---Y-I-:)1:-+-(-I2---Z-l-:-)'

=; J9+ 16+ 144 = 13

d.r.'s of PQ are x, -XI' Y, - Y,. I, - I,

i.e., 3,4, 12

3 4 12 :. d.c.'s of PQ are 13' 13' 13

[Dividing each by J3' +4' + 12' =; 13] Example 33. P, Q, R, S are the points (- 2, 3, 4) (- 4, 4. 6). (4, 3, 5) and (0, I, 2). Prove by projections that PQ is at right angles to RS.

SoL The four points are P (-2, 3, 4), Q (-4, 4, 6), R (4, 3, 5), S(O, I, 2).

. . d.r.'s of RS are

o - 4, I - 3, 2 - S I Using Xl - X" Yl - Y" X, - I,

or -4, -2,-3

or 4, 2, 3

Dividing each by • .J16+4 + 9 =.,fi§ 423 the actual d.c.'s of RS are .J29' .J29' .J29

. . Projection of PQ on RS

4 2 3

= .J29 (- 4 + 2) + .J29 (4 - 3) + .J29 (6 - 4)

(Using l(x, - x,) + m(y, - YI) + nez, - II)]

I

= r= (- 8 + 2 + 6) :: 0 ..... 29

.. PQ..l RS.

~Locus represented by equatioDI-Remember

U rheberrec htl lch gesc h utztes Iv! ateri"

Direction Cosines and Projections

53

~ (J) Eqn. of a plane II to YZ-plane and at a distance (I from it isx=« Eqn. of a plane II to lX-plane and at a distanc-e b from it isy=b Eqn. of a plane II to XY -plane and at a distance c from it is z=c The equation of a plane II to any co-ordinate plane is missing VII riable=consta nt,'

Again putting a, b. C eo O. in the above, the equations of the Y=, ::x and .\}I planes are x '" 0, y '" 0, z '" ° respectively,

(2) Equations of x-axis are y = 0, z = °

Equations of j-axis are z = 0, x "" °

Equations of s-axis are x = 0, y '" °

(3) Again. an equation only in one variable represents a system of planes (real or imaginary) all of them II to missing co-ordinates plane. Further an equation in two variables only represents a cylinder whose axis is II to the missing co-ordinate axis and finally, an equation in three variables represents a surface.

urhcbcrre<;htlich gcschutztes Material

3

The Plane

Plane (DeC.). A plane is a swface sucb that if any two points OIl it are taken, then the line joining them lies wholly in the plane.

Art. 1. General Form

Jar Prove tha' general equation of fint degree In. X, y •. z .represeats a plane. (Gar/rwaJ 1988; Agra 1983; PWljab 1981)

Let the first degree equation is x; " z be

ax + by + cz + d = 0 ... (1)

Let P(x,. ',. %,), Q(.1'l' 'l' %z} be any two points on the locus given by (I) and R be ANY point on the line PQ .dividing it in the ratio m. : ml, Then

ax, + by, + cz, + d ... 0 (2)

ax1 + by! + C%Z + d = 0 (3)

MUltiplying (2) by ml and (3) by m. and adding, we get

tl(m!x, + m,xz} + b(m!" + m,Yl) + c(m1Z1 + m,%z) + a{m. + ml)'" 0

Q (m,xl + rnz.1',) + b (~Yl + rnz,,) + c(nr'Zl + rnzz,) + d == 0

~+rnz ~+~ ~+~

or

wbich clearly shows that the point

R (mix, +rnzx" m"2 +rnzY" m,%z +II7zZ,.)"

m, + ~ mt +1I7z m, + II7z

lies on (I) for all values of m" mr

Thus we have shown that if p(.1',. Y" %,). Q(Xl' YI, Zl) lie on (I), then every point R of the line joining P and Q lies on (I).

i. e., the line PQ lies wholly on (I).

Therefore the surface represented by (I) is a plane surface.

Thus a general equation of first degrecin x, " Z always represents a plane,

Cor. (One .polnt form) : To show tbat the equation of Iny plane tbrouRb (x" y,.z,) Is

A(x - I.,) + B(y - Y,) + C(z - z,) - 0

54

U rhcberrec hit lch ge5c h "Illes M ateri"

ThePlsne

55

Let the equation of required plane be

Ax + By + Cz + D = 0 ... 0>

Since this passes thro' (x.' Yl, %,) :. Ax. + By, + ez, + D = 0 Subtracting (2) from (I), we have

A(x - Xl) + B(y - Y,) + C(z - z.) - O. which is the required equation.

Note. The general equation of the plane is Q)C + by + cz + d ~ 0 or

in x, y, z represents

-: any first degree eqn

a plane

.,,(2)

1M. Imp.1 IC.T.M.,

a b' c

-x+-y+-:+1 '" 0 or a'x+b'y+c'z+ 1::: 0 d d d

which has three independent constants. Thus a plane must always satisfy three conditions.

Example 1. Show that the plane ax + by + cz + d = 0, divides 1M'

join of (XI' YI• z.) and (xl' Yl' z:) In the ratio

tn',+by,+cz,+d. aJcz +byz +cz2 «d

SoL Let the plane

ax + by + cz + d = 0 ... (1) divide the join of P(x,. Y" z,) and Q(xl' Yl' Z2) in the ratio k : I at R.

Then co-ordinates of R are

(10:1 + Xl, .t:vz + Y'. kzz + z, )

k+l k+1 k+l

Since R lies on the plane (I),

.. a (hl +x, l- (kyt + y')+c(kz1 +z, )+d = 0

k+1 k+1 k+1

or a(hz + x.) + b(lcyz + y,) + c(kzz + ::,) + d(k + I) = 0

or ..t(aJcz + by: + czt + d) = - (ax, + by, + cit + d)

p----

(K,. y,. Z,

R

--- .. --

k: 1

a (x,. y,. l,l

k =: _ ax, +by,-czl +d .

.. at2 +byl +CZ2 +d

Example 1. Find Ihe ratios in which the co-ordinate planes divide the line joining (-2, 4, 7) and (J, -5, 8~

SoL The co-ordinates of the point R which divides P(-2, 4, 7) and

Q(3, -5, 8) in the ratio k : 1 are .

(3k-2, -?k+4, 8k+7)

k+1 . k+1 k+1

U rhebcrrec htl ich gcsc h utztcs ~ I aterla

56

. Golden Solid Geometry

(I) If it lies on the YZ-plane Le. on x = 0, we haye 3k-2 2

--=0 or k=-

k+1 3

i.e. the YZ.plane divides PQ internally in the ratio 2 : 3.

(;1) If R lies on the ZX-plane i.e., on y = O. we have

-Sk+4 =0 or k=~

k+1 S'

Thus ZX-plane divides PQ Internally in the ratio 4 : S. (iii) If R lies on the XY-plane i.e: on z = 0, we have

Bk+7 7

--=0 or k=--

hi B

Thus XY -plane divides PQ externally in the ratio 7 : B.

Art. 2. Intercept form

(a) To find Ihe equatlon of the plane which cuts off Intercepts a, b,

e on Ihe axes (Agra 1986; K.U. 1982; Bihar 198.1;

Ranchi 1980, 83, 86)

Let the equation of the plane be Ax+By+Cz+D = 0 ... .(1) Let the plane meet the axes in P, Q.

R such that

OP '" a. OQ = b, OR = c.

Then the co-ordinates of P, Q, R are P (a, 0, 0) X

(a, 0, 0), (0, b, O) and (0, 0, c). Since P(a, 0, 0) lies on the plane (I)

D :. A· Q + D = 0 or A=--. a

Similarly, the plane passes thrO' Q(O, b, 0) and R(O, 0, c).

:. Bb + D .. 0 or C . c + 0 '" 0

.. D D

which gives B=-b"' C=-~

Putting these values of A, B, C in (I), we have

D D 0

--x--:- y-- %+D=O

Q b c

x y z

or cancelling - D'':-'+-b +- '" I

Q 'c

which is the required equation.

U rheberrec htl lch gesc h utztes Iv! ateri a

TheP/sns 57

Remember. Equation of the plane in the intercept form is

x + y + l =1

intercept on x-axis intercept on j--axls intercept on s-axis

Note. For problems relating to the intercepts of a plane on the axes or relating to a plane meeting the axes in A, B, C, we take the equation of the plane in the intercept fonn

~+L+':'=1. abc

(b) Reduetion oUhe general equation oUhe plane to the intercept form

Let the plane be ax + by + cz + d = 0 ... (1) (General form) This can be written as

.<u+by+cz=-d

or

ax +L+ cz .:::) -d -d -d

(On dividing by -dJ

or

~+L+~=l -d -d·-d

abc

x y z

which is of the form -+-+- = I. (intercept form)

ABC .

Hence the lengths of the intercepts of (I) on the axes are

-d -d -d

-, -b ' - respectively.

a c

Note. To find the intercept 011 x-axis, put y "' 0, Z "'". 0 in (I) ..

d

Thus ax + 0 + 0 + d - 0 or .l =--; is the lntercept on x-axis,

Similarly the intercepts on y and a-axis can be obtained.

Example 3. Does the point (4, - 6, 0) lie on the plane which intersects the positive x, y and z-axes at distances 2, 3, 5 units respectively.

(Calcutta 1984) Sol. Equation of the plane which intersects the positive x. y and zaxes at distances 2, 3, 5 units respectively is,

!.+l.+':'::: I 2 3 5

Putting x = + 4, y = - 6, Z "" 0 in L.H.S.

4 -6 0

L.H.S. ="2+3+"5

=2-2+0

'" 0 (:;t I, the R..H.S.)

Hence thepoint (4, - 6, 0) does not lleon (I).

... (1)

U rheberrec htl lch ge5c h utztes Iv! ateri"

58

Golden Solid Geometry

Example 4. A variable plane moves so thaI the sum of reciprocals of its intercepts on the three co-ordinates axes is constant. Show thai it passes through a fixed point.

SoL Let the variable plane be

~+L+':'=I abc .

so that its intercepts on the axes are a, b, c.

Now we are given that

1 I 1 I

- + - +- = constant = -k (say) abc

... (1)

k k k -+-+-=1

.• Q b c

equation (2) shows that the point. (4; k, k) satisfies plane (I) I.e. plane (I) passes through. the fixed point (4; k, k).

Example S. A variable plane passes through a fixed point (a, b. c) and meets the co-ordinate axes in A, B, C. Show that the locus of the point common to the planes through A, B, C parallel to the co-ordinate planes Is abc -+-+-= I.

x y z

... (2)

(Utkal 1982; M,D.U. 1984; Bhopal 1985) [Imp.]

SoL Let the equation of variable plane be

... {I)

[Here we have taken p. q, r because a, b, c are given] Since it passes througb (a, b, c),

abc -+-+-= 1 p q r

The plane (I) meets the axes in A, B, C so that

OA = p. OB = q, OC = r. Z

Now equation of plane thro'

A(p, 0, 0) and II to yz plane is

x '" P ... (3)

Similarly the equations of the planes thro' B(O, q, 0), and C(O, 0, r) and II to ZX and XY.planes respectively are

y = q (4)

and z = r (5)

The planes (3), (4), (5) meet in the point P, where

... (2)

y

U rhcberrec htl lch gesc h utztes M ateri a

The Plane

59

x '" p, y '" q, Z = r ... (6)

To find the locus of P, we have to eliminate p, q. r from (2) and (6). Thus the required locus is obtained by putting values of p, q, r from (6) in (2).

Th '_" I . abc

.. e requirea oeus IS -+-+-= I.

x y z

Example 6. A plane meets the co-ordinate axes at A .• B. C such that the centroid 'of the triangle ABC is the point (a. b, e). Show that the equation of the plane is

x . Y z -+-+-=3. abe

(K.U. 1986; Agra .1984; Lucknow 1984;

Kanpur, 1983, 81; Panjabi 1985) Sol. Let the equation of the plane be

or

~+L+.E.=1 P q r

It meets the axes in A (p, 0, 0), B (0, q, 0), C (0, 0, rl. The centroid G of the MaC is

(P+O+O O+q+O o+o+r) 3 '-3-"-3-

(f, ~, ~)

But it is given to be (a, b. e).

l!.. '" a which gives p '" 3a 3

!!... '" b which gives q '" 3b 3

r

'3 '" c which gives r '" 3c

Putting these values of p, q; r in (I), we have

, .. (I.,

x . Y Z

-+-+- = I 3a 3b 3c

x y z

or -+-+- = 3 abc

which is the required equation of the plane.

Art. 3. Norma.1 form

To fiDd the equation of a plane in terms ofp, the length of a perpendicular from tbe orialn on the pia De, and .. m, n the direction cosines of this perpendicular, (Calcutta 1985; K.U. 1985; VeJ1k 1984)

U rheberrcc htl lch gesc h utztes Iv! ateri"

60

Let ABC be plane and let ON be the .1 from 0 on the plane, so tbat ON = P and d.c.'s of ON are I, m, n.

Lei P(x, y, z) be any point on the plane. Join OP and PN. Then ON .1 PN.

[ ... if a' line is .1 to Ii plane, then it is .1 to every 51. line in the plane.] .. ON = projection of OP on

ON

Golden Solid Geometry

z

y p g J(x - 0) + m(y - 0) + n(z - 0)

I l(x2 - XI) + m(Yl- YI) + n(Zl- ZI)

or Ix + my + nz = p

which is the required equation.

or

formula [C.T.M]

Another form of this Art.lde

Establish the equation of a plane in the form

x cos a + y cos ~ + Z cos y - p.

Here 1 = cos c, m= cos 13, n = cos y.

Note I. p is always positive.

Note 2. An equation lx + my + lIZ + P is in the normal form if (i) (co-efJ. of X)l + (co-efJ. of y)Z + (co-efJ. of z}l "" 1

and (11) cons I. term on tire R.H.S. is +ve,

Art. 4. Reduction of general form to normal form

To reduce the equation Ax + By + Cz + D '" 0 to tbe normal form Ix + my + DZ - P

The given equation is Ax + By + Cz + D = 0 (1)

and the nonnal ·form is lx + my + nz - p = 0 ' (2)

[We are required to find I. m. n and p in terms of A, B, C, D.] Comparing the co-efficient in (I) and (2), we have

I m n -p

-:-=-=-

ABC D

fe.T.M)

-pA -pB -pC

.. I=O,m=o,n=o'

Since I, m, n are the d.c. 's of the normal to the plane

or

or

urhcbcrre<;hllich gcschOtztcs Mal<:ria

The Plane 61

and

Case I. If D is +ve, then as p is always +ve, we have

D D

p= .fA2+B2+C2= ~r.A2

-A -8 -C

•• /=_. -- m=--- n=-'--

.Jr.A~ • ..Jr.A1' .Jr.At·

.. The normal fonn of (I) I Putting these values in (2)

ABC -D

---- x----· y--.-- z----

"r.A2 "r.A 2 ""'LA 1 - "LA 1 .

is

Case n. If D is -ve, then since p is always +ve,

-D -0

p= =--

JA1+Bl+C1 if-A2•

ABC

1=--- m==--- n=---

.Jr.A 2 ' .Jr.A 1 ' .Jr.A 1 •

.. The nonnal fonn of (1) is

ABC -0

--- x+--- y+--- z=--- .

.Jr.A 1 .Jr. A 1 .Jr. A 2 "r. A 1

Rule to reduce general rorm to normal rorm :

(I) Transpose the constant term to R.H.S and make it positive (if it is not $0 already).

(i/) Divide throughout by .fA! +B2 +C1 i.e. by

~(co-eff. of x)2 + (co-eff. ofyi + (co-eff. of z)2 Cor. Direction cosines of the normal to a plane

The d.c.ts of normal to the plane

Ax + By + Cz + 0 == 0

IImp·1

are ±_A_ ±_B_ ±_C_ .Jr. A l' "r. Bl' .Jr. C1 ' Thus d.c.'s are proportional to A, B, C.

~ Hence, the direction cosines of tbe normal to 8 plane are proportional to

oo-eff ofx, co-eff. of y. co-eff of z, [M. Imp.] [C.T.M] I" the equation of plane.

i.e., these give d.r.'s of normal to the plane.

[.+ sign if 0 is -ve] - sign if Dis +ve

Cantlon. Co-efficients of X, y, z in the equation of the plane are d.r, 's ofnonnal to the plane and not that of the plane. {d.r.'sare related to a line only.]

U rheberrec htl lch gesc h utztes Iv! ateri a

62

Golden Solid Geometry

Example 7. If the axes are rectangular and P is the point (2, J, -I), find the equation to the plane through P at right angles to Ol". (Ruhelkhand 1983)

Sol. Here 0 is (0, 0, 0) and P is (2, 3, - 1)

., d.r.'s of OP are 2 - 0, 3 - 0, - I - 0, i.e. 2, 3, -I. These must be the

co-efficients ofx. y, z in the equation of the plane which is at right angles to Op,

:. Let the equation of the plane be lx + 3y - z + K ~ 0 ... ( I)

Since it passes through P(2. 3, - I),

:. 2(2) + 3(3) - 1(-1) + K = 0 or K = -14 From (I),lx + 3y - z - 14 = 0

which is the required equation of the plane.

Example 8. 0 is the origin and A is the point (a, b, c). Find the direction cosines of the join of OA and deduce the equation of the plane through A at right angles 10 OA. (Ruhelkhand 1983)

Sol. 0 -+ (0, 0, 0) A -+ (0; b. c)

d.r.'s of OA are a - 0, b - 0, C - 0

i.e., a, b, c

abc

:. d .. c.'s of OA are i'i.o~' ~r.a' ' ~"i.Q~

where 'i. Q2= 02 + b2 + c2

Any plane through A(a, b, c) is,

A, (x - 0) + B, (v - b) + C, (z - c) = 0 ... (1)

where A .. BI• CI are the direction ratios of the normal to the plane.

Since (I) is given to bel. to OA,

AI BI CI

.. --0- = ~-b-' --c-

~r.Ol J'i.ol ~'i.ol

Hence (I) becomes

a(x - a) + b(v - b) + c{:! - c) = 0

i.e .• &=,.;,=s.

b

c

~ ax + by + cz = a1+ tr+ c! which is the required equation.

Example 9. Find the equation of the plane tha! passes through (2, ~3, I), and is perpendicular to the line joining the points (3, 4, -I) and

(2, -I, 5). (Agra 1987 Type; D,U. 1981)

Sol. The equation of any plane thro' the point (2,- 3, I) is

A(x ~ 2) + B(v + 3) + C(r - I) '" 0 ... (1) I One point form Also the d.r.'s of the line joining P(3, 4,-1) and Qf2, -I, 5) are proportional to

2 - 5, - 1 - 4, 5+ I I Using .\"1- XI' )11 ~ Y,. :2 - J"

or

1, 5, -.6

U rheberrec htl lch gesc h utztes M ateri a

The Plane

63

Since the plane (I) is 1. to PQ, :. the normal to the plane (I) whose d.r.'s are A, B, C (viz. co-effs. of x, y, i) is 1110 PQ whose d.r.'s are I, S, -6 (proved above)

ABC

:. 1 = 5" = =6= k (say)

Putting the values of A, B, C in (I), we have

k(x - 2) + 5k(v + 3) - 6k(z - I) = 0

or x+5y-6z+19=O

which is the required equation of the plane. Second Method :

Here P is (3, 4, -I) and Q is (2, -I, 5)

d.r.'s of PQ are 2 - 3, - I - 4, 5- (-I) I Using x. - x,. Y}- Y" t1- t,

or -I, -5, 6

or I, 5, -6

These 1, 5, -6 must be the co-efficients of x, Y, z in the equation of the plane which is at right angles to PQ.

:. The equation of the plane is

x+Sy-6t+k=O ... (1)

Since it passes through (2, -3, I)

:. 2 - 15 - 6 + k = 0 or k '" 19

Putting k '" 19 in (I), the equation of the plane is

x + 5y - 6z + 19 = 0,

~ El8.mple 10. (a) A variable plane is at Q constant distance pfrom the origin and meets the axes in A, D, C. Through A, B,. C, planes are drawn parallel to the co-ordinate planes. Prove that the locus of their point of intersection is X-I + yl + rl = [Fl. [V. Imp.]

(Ravishankar 1984.; Vikram 1984; Kanpur \988; Rajputana 1983; Garhwal 1983; K.U .. 1986, 83) (b) A variable plane which remains at a constant distance 3p from the origin CIlIS the co-ordinate axes in A, B. C. Show that the IOCIlS of the centroid of the triangle ABC is x-l + y1 + rl = p-1. [V. Imp.) (Indore 1984; Madurai 1982; Kanpur 1984; Ravishankar 1981; Avadh 1983; .Meerut 1985, 83) (c) A variable plane is at a constant distance p from the origin and meets the axes in ;I, B, C. Show that the locus of the centroid of the tetrahedron DABe is ;rl + yl + rl = 16[F1, [Imp.] (Agra 1988, 86, 84, 82; Bhopal 1984; Meerut 1983; Lucknow 1982;

M.D.U. 1985; Raj. 1981) Sol. (a) Let the variable plane at distance p from the origin be

Ix + my + nz '" p ... (J) (Normal form)

where I, m, II arc the d.c. 's of the norm ill to the plane. The plane '( I) meers .e-axis where putting Y'" 0, z '" 0, we have

U rheberrec htl lch ge5c h utztes Iv! ateri a

64

Golden Solid Geometry

Ix =p or x=l!..

I .

Hence co-ordinates of A are (7' 0, 0) Similarly the points Band C on the y-axis and z-axis are

(0, ~, 0) and (0, 0, ~)

Equation of the plane thro'

A (7,0,0) and II to YZ plane is

x=.!!.. (2')

I ... ,

z

C (0, 0. .e. ) II

A X

(.e.. D. 0)

I .

y

Similarly the equations of the planes thro' B (0, ~, 0) and C ( 0, 0, ~)

and respectively II to ZX and XY planes '

are Y =.E... ... (3)

m

and z =E..

n

The three planes (2), (3) and (4) meet in the point

p [x::. E., y .r., z::. P.].

I m n

. .. (4)

From this we have 1=£', m=£', n=E,

x y z

Since I, m. n are the dc . 's, .. f1 + m1 + rr = I

p2 pl pl

-+-+- I

Xl yl %,1 =

or

I I I

m -+-+- -

x2 . y1 Zl p2

or X-I + y1 + z-1 ""' pol

which is the locus of P.

(b) Let the variable plane at a distance 3p from the origin be

lx + my + .nz '" 3p ... (1) I Normal form

This meets the x-axis in A where putting y = z= 0, we have

Ix ::. 3p or

3p x =!'

U rhcberrec htl lch gesc h utztes M ateri"

ThePfane

65

Thus A is e:, 0, 0).

Similarly the plane meets y-axis and z-axis in the points

B (0, ~. 0) and C (0, 0, 3:}

Now if G (x. y, z) be the centroid of the MBC, 3p +0+0

.l'= I =E.

3 I

0+ 3p +0

r= m =E.

3 m

0+0+3p

z= __ ...:.n:,_ l?

3 n

... (2)

Moreover since I, m, n are the d.c.'s (of the normal to the plane)

.. P + nil + n2 = I. ... (3)

Putting the values of I. m. n from (2) in (3), we have

p2 p2 pI

-+-+- == 1

x2 y2 Z2

or

J 1 I

-+-+- -Xl y2 Z2 - pI

,%"2 + yl + r2 = p-2 which is the required locus of G. (e) Proceeding as in pan (a), we have

A ( f. 0, 0); B (0, ;;, 0); C (0, 0, ;)

Let (x. y, z) be the centroid of tetrahedron OABC.

0+7+0+0

X=--'---

or

4

urhcbcrre<;htlich gcschutztcs Maleri"i

"66

Golden Solid Geometry

0+0+£+0

y= m

4

O+O+O+P...

z » .....::n~

or

4

x= ~,y= f'm,Z=:"

From Ibis we have / = ..!!...., m '=" .J!...., n '=" J!....

4x 4y 4z

Since 1, m, n are d.c.'s :. P +' "r + til = t

Putting the values of l, m, n, we have

p2 pI pI --+--+-- = I

t6xl 16yl. 16:'

I I 1 16

or -+-+- =-

x2 y' Z1 p'

or x-1 + y2 + z-2 = 16p-2

which is the required locus of centroid of tetrahedron.

Second Metbod : .

Let the variable plane be

~+.l+':'= I abc

meeting the co-ordinate axes in A(a. 0, 0); B(O, b, 0) and qo, 0, c). p =.1 distance of 0(0, 0, 0) from (1)

... (1)

i.e.

-I

S . 2

quanng p = I I 1

-+-+-

02 b2 cl

1 1 I I

-+-+-=- (2)

.. al bI c2 p2 ...

Let (x, y, z) be the centroid of the tetrahedron OASe where 0(0, 0, 0); A(a, 0, 0), B(O, b, 0). C(O, 0, c)

Urbeberrechtlich gcschQtztcs Maleria

ThePlsne

67

I Using

(~+~:~+~,~+h:~+~ ~+~:~+~)

. . a = 4x, b = 4y, c = 4z

Putting these valuse of a, b, C in (2), we get

I 1 1 1

--+--+-- --

16x1 J6y2 16z2 - pI

or ,r-I + yl + rl = 16p-1

which is the required locus of centroid of tetrahedron OABC.

Art. S. Three point form

To find the equation 01 tbe plane passing through tbreepoints (XI' YI' ZI)' (x" YI• ~). and (x". YJ• zJ·

(Madras 1.983; Mirhila 1982; Agra 1982; Kanpsr 1983; Bhogalpur J 983; Ran chi 1986) Let the equation of the plane be

Ax + By + Cz + D = 0 ... (1)

Since it passes thro' (XI' YI, ZI)' (Xl' Yl, Z2)' and (Xl' 'vI' z)

•.• Axl + BYI + Czl + 0 '" 0 (2)

Axl + BYI + Cz1 + D= 0 (3)

Ax) + By) + Cz, + D '" 0 (4)

Eliminating A, B, C, 0 from (I), (2), (3) and (4) with the help of determinants, we have

x Y Z
x( YI z( =0 C.T.M
Xl Y2 ZI
XJ YI ZJ which is the required equation of the plane,

Cor. Condition for four given poln.ts to be coplanar

The four points .(.1:1' YI, z), (Xl' Yl, .Z2), (XJ, YI, Zl) and (x4, Y4, z.) will be coplanar if a plane passing through any three points passes through the fourth point ..

Now the plane thro' (x" YI' Z2)' (x" YI, Zl)' (x.' Y., z.) is

x Y Z
Xl YI Zl =0
.x) Y) Z)
x. Y. z.
U rhcberrec htl lch ge5c h utztes M ateri" 68

Golden Solid Geometry

XI YI %1
X Yz Zz =0
I
. Xl Y, Z,
X~ Y~ Z4 which is the required condition.

Note. II.mp.J. In numerical problems., it is more convenient to follow Ihe method

given below in solved example 11. .

Example II. (a) Find the equation of the plane through the three poinJs (0, J, f), (I, J, 2), (-1. 2, -2).

G' (b) Show thaI the points (0, -I, 0), (2, J, -I), (I. J, f), (3, 3, 0) are coplanar. (Bihar 1981; Agra 1985)

(c) Show that the po.inls '0. -I. -I), (4, 5, I), (3, 9, 4) and (-4,. 4, 4)

lie on a plane. (Meerut 1982)

(d) Prove tha: the line joining the points (1, 2, -I), (3, 4, 1) intersects

the line joining the points (7, 0, 6), (2, 5, I). [Y. Imp.]

Sol. (a) The given points are (0, 1, I), (I, 1,2), (-1,2, -2). Equation

of any plane thro' (0, I, I) is

A(x - 0)+ B(y - 1)+ C(z - I) = o. I u .. Sing A(x -X.I) + B(y -!l)

... (1) + C(z - %1) - 0

one; point form

Since it also passes thro' (I, I, 2) and (-1, 2, -2)

. . A + 0.8 + C= 0 ... (2)

and -A+B-3C"0

or

A-B+3C=0

Solving (2) and (3) by cross-multiplication, we have

.. :(3)

ABC

--=.-. -::::--- -

0+ 1 1-3 -1-0

A :: 8 =.£ = k (say) 1 -2 -I

. . A = Ie, B = - 2k, C :: - k.

Putting these values of A, 8, C in (I), we have

fa - 2k(y - I) - k(z - I) .. 0

or X - 2(y- I) - (z - I) ... 0

or . x-2,)'-z+3"0

which is the: required equation of the plane.

(b) Let us first find the equation of the plane through three points (0, -I, 0), (2, I, -I) and (1. I, I).

Equation of any plane thro' (0, - I, 0) is A(x-O) ... 8(y + I) + C(z- 0) ~ 0 ... (1)

or

l One poinl form

U rhcberrec htl lch gesc h utztes M ateri"

The Plane

69

it passes thro' (2, 1, -I) and (I, I, I), 2A + 28 - C = 0

... (il) ... (iil)

A + 2B + C = 0

Solving (it) and (iii) by cross-multiplication, we have

ABC ABC

--=--=-- or -=-=-.

. 2+2 -1-2 4-2- 4 -:J 2

Putting these values of A, B, C in (I), we have 4x - 3(y + I) + 2z = 0

or 4x - 3y + 2z - 3 = 0 ... (iv)

The four given points will be coplanar if the fourth remaining point (3, 3, 0) lies on the plane (iv)

i.e. if 4(3) - 3(3) + 2(0) -3'" 0

or if 12 - 9 -3= 0 or if 0 = 0 which is true.

Hence the four points are coplanar. (e) Please try yourself,

(d) The four given points are P(2, 2, -I), Q(3, 4, 2), R(7, 0, 6) and 8(2,5, 1). We are to prove that the line joining P, Q intersects the line joining R. 8. The line PQ intersects RS, if the points P, Q, R. S lie on the same plane (.: two intersecting lines always detennine a unique plane). Thus we are to prove thai all the four given points are coplanar.

Now the equation of any plane thro' P(2, 2, -I) is A(x - 2) + B(y - 2) + C(Z + I) = 0

It passes thro' Q(3, 4, 2) and R(7, 0, 6) if A+2B+3C=0

SA - 28 + 7C = 0

.... (/)

and

... (iI) ... (iil)

Solving (ii), (iii) by cross-multiplication,

A 8 C

or

14+6 15-7 -2-10

ABC ABC

-=-=-- or -=-=-.

20 8 -12 5 2 -3

Putting these values of A, B, C in (I), we have the equation of the plane thro' P, Q, R as

5(x - 2) + 2(v- 2) - 3(z + I) = 0

or 5x + 2y - 3: - 17 = 0 ... (iv)

The four given points will be coplanar if the fourth point S(2, 5, 1) lies on (iv) i.e. if

5(2) + 2(5) - 3(1) - 17 = 0

or if 10+10-3-17=0 or if 0=0 which is true.

Hence P, Q, R. S are coplanar and :. PQ intersects RS.

Example 12. Find the equation to the plane through the points

(I, J, 0), (I. 2, I), (-2, 2. -l). (Meerut 1986)

U rhcberrec htl lch ge5c h utztes M ateri a

70

Go/den Solid Geometry

Sol. Please try yourself, exactly as Example 11.

[Ans. 2x + 3y - 3z 0; S1 Example 13. Find the direction cosines olatty normal ./0 the plane passing through the points (0, -I, -I), (4, S, I), (3, 9, 4), and (-4, 4, 4).

. (Ranchi 1982)

SoL The equation of plane through first three points is

Sx - 7y + liz + 4 = 0 ... (1) I Find yourself as in Ex. II Clearly the point (-4, 4, 4) lies on il.

Now, d.r.'s of normal to (I) are 5,-7, II

.. d.c.'s of nonnal to (I) are

5 -7 II

JSz +(_7)1 + Ill' JS2 +(~7)1 + 1 e' JSl +(_7)2 + liZ

5 -7 11

t.e., .[19S' .JI9S' .,ff9S'

Exa.mple 14. If from the point P(a, b. c). perpentliculOl"s PL, MP be drawn (0 .rz and ZX planes, find the equation of the plane OLM

- (D.U. 1983)

Sol. The co-ordinate of L. the foot of .1 from Pea, b, c). on YZ-plane (where x = 0) are (0, b, c) .

. The co-ordinate of M, the foot of .L from pea, b, c) on ZX-plane (where y= 0) are (a, 0, e).

Thus we have L(O, b. e), M(a, 0. c) and 0(0, 0, 0).

Now equation of any plane thro' 0(0, 0, 0) is A(x - 0) + 8(y - 0) + e(z - 0)= 0

oc ~+~+~-O

it passes thro' L(O, b, e),

... A.O + b.B + e.C '" 0

Since (I) passes thro' M(a, 0, e),

. . .. aA + O.B + cC = 0

So.lving (il) and (iil) by cross-multiplication, we have

l One point fonn .... (1)

... (iI)

... (ii/)

A

B

c

be-O = ac-O = O-ab ABC

-=-=-

be ac -Db'

Putting these values of A, B, C in (I), we have hex + aey - ab:. = 0

':'+1:_':' = 0

or abc

which is the required equation of the plane OLM.

or

U rhcberrec htl lch ge5c h utztes M ateri"

The Plane

71

Eumple 15. The plane passing through the points (2. 0, 2), (6, 1, I), and (4, 2, 3) meets the co-ordinate aru at A, B, C. Find the centroid of t1 ABC. (K.U. 1985) SoL Any plane through (2, 0, 2) is

A(x - 2) + B(v - 0) + C(Z - 2) = 0 ... (1)

It passes through (6, I, I) and (4, 2, 3)

.. A(6 - 2) + B(I - 0) +C(I - 2,= 0 (2)

and A(4 - 2) + 8(2 - 0) + C(3 - 2) = 0 (3)

Eliminating A, 8, C from (I), (2), (3)

Determinantly, we get

x-2 y z-2

4 -I =0

221

::::) 3(x - 2)- 6(v) + 6(: - 2) '" 0

::::) z-2-2.l'+b-4=O

::::) x - 2y + 2z ., 6

z y z

::::) "6+ -3+"3= I

:. A _. (6, 0, 0); .B _. (0, - 3, 0); C _. (0, 0, 3) Let centroid of t1ABC be (11, ~, 1), Then,

6+0+0 a=---=2

3

0-3-0

~= -3-=-1

0+0+3

y= --= 1

3

Hence the centroid is (2, -I, I),

Example 16. Show thol (-1, 4, -3) .is the circum centre of the triangle formed by the points (3, 2, -5), (-3, 8, -5) and (-3, 2, J).

SoL P( -1, 4, -3) is the circumcentre of the triangle formed by the points A(3, 2, -5}, 8(-3, 8, -5) and C(-3, 2, I) if

(I) PA .. PB = PC

and (ii) P, A, at C are coplanar I Note

Now PA = J(3+I)l +(2_4)' +(-5+3)~ = • .li6+4+4 ",,2./6

PB '" J(-3 + l)~ +(8-4)~ +(-5 + 3)1 ::: J4 + 16+4 -= 2./6

PC = J(-3+ 1)1 +(2 _4)2 +(1 +3)1 = J4 +4 + 16 = 2./6

U rheberrec htl lch gesc h utztes Iv! ateri a

72 Golden Solid Geometry

.. PA= PB '" PC and hence condition (I) is satisfied.

Again, equation of any plane thro' A(3, 2, - 5) is

a(x - 3) + b(y- 2) + c(z + 5) '" 0 ... (1) [One point form] If it passes through 8(- 3, 8, -5) and C(-3, 2, I), then

- 60 + 6b + O.C = 0 or a - b + O.c = 0 (2)

and - 6a + O.b + 6c = 0 or a + O.b - c = 0 (3)

abc

Solving (2) and (3) - = - = -

I I I

.. Putting these values of a, b, c in (I) I(x - 3) + l(y - 2) + l(z + 5) = 0

or x+y+z==O ... (4)

The point P(-I. 4, - 3) lies on (4), if

- I + 4 - 3 = 0 or if 0 '" 0 which is true.

Hence the points P, A. B. C are coplanar. Thus the condition (;0 is also .satisfied.

Hence P is the circumcentre of AABC.

Here circumradius = PA[= PB == PC] ""2./6.

Example 17. Show that the six planes each passing through one edge of a tetrahedron and bisecting the opposite edge meet in a point.

Sol. Lei A(x" Y1 • .!). B(x2' Y1, %1)' C(xj• Y,. Z3)' D(x,. Y" z.) be the

vertices of the tetrahedron ABCD.

Let L, M, N, P, Q, R, be the midpoints of the various edges as shown.

Now consider the plane ABL through the edge AB and passing thro' L (i.e. b~cting the opposite edge CD).

. (Xl + x., Yl +x'. z) +z •. )

L IS 2' 2 ,. 2

Since line AB lies on this plane. so

c (x,. y" zJ

p(XI+-'"l Y,·+Yz z,+%z.)

2 ' 2 ' 2

the mid-point of AB also lies on this plane. Again, sinceP and L lie in this plane. .. the line .PL Wholly lies in this plane and so the mid-point of PL viz.

° (XI +X1 + X 1+-'", YI+Y2+Yj+Y, ZI+Zl+Zl+Z4)

4 • 4 ' 4

also lies on this plane.

Continuing the same argument we can show that G lies on other five planes also which pass thro' the other edges and bisect the opposite edges.

U rhcberrec htl lch ge5c h utztes- M ateri a

The Plane

73

Hence G is a point common to all the six planes. In other words the six planes (each passing through one edge and (bisecting the opposite edge) meet in a point G.

Hence the result..

Example 18. Find the equations 0/ the two planes through the points (0, 4, -3), (6, -4, 3) which cut off from the axes intercepts whose sum is zero, (Agra 1981)

SoL Let the equation of the plane be

':'+2::+£=1

abc (1)

We are given that a + b + c '" 0 (2)

(1) passes through the points CO, 4,. -3) and (6, -4, 3).

and

o+~_2 = I

b c

643

---+-

abc

Adding (3) and (4), we get

6

-=2 or a'" 3

... (3)

... (4)

a

4 3 From (3), "b--;; == I

and from (2), b + c = - D = -3 . . b "'....:3 - c ... (5)

Putting this value of b in (3), we have

4 3

'" 1

or

-3-c c

4 3

--+-+1 c+3 c

=0

or 4c + 3c + 9 + C' ... 3c '" 0

or C'+lOc+9=0

or (c + I) (c + 9) '" 0 which gives c.=-l or -9.

From (5) when c '" - 1, b = -3 + [ '" -2

when c=-9,b=-3+9=6.

Putting these values of D, b, c viz.

a = 3, b = -2, c '" -1 and a = 3, b = 6, c =-9 in (I) we get

~+L+2...= [and .:. +l. +2... = I

3 -2 -1 3 6 '~9

i.e. 2x - 3y - 6z = 6 and 6:c'" 3y - 2z '" 18

which are the required equations.

U rheberrec htl lch gesc h utztes Iv! ateri a

Você também pode gostar